Anda di halaman 1dari 46

Review for NCLEX-RN Examination 301-400

--> QUESTION NUMBER _ 301 _ about (MC) QUESTION: "Which of the following statements would indicate that the parents of a child being treated with antibiotics for an ear infection understand the reason for a follow-up visit after the child completes the course of therapy?" CHOICES ( X ) a.) "Her hearing needs to be checked to see if the infection has done any damage." ( X ) b.) "The doctor wants to make certain she has taken all the antibiotics." ( O ) c.) "We need to make sure that her ear infection has completely cleared." ( X ) d.) "She needs to get another prescription for second course of antibiotics." RATIONALE: Because ear infections are sometimes difficult to treat, determining if the antibiotic has resolved the infection is essential. If the client is not rechecked, it will be difficult to determine if another infection is a continuation of a previous infection or a separate new infection. Although studies may be done to determine if an infection has impaired the child's hearing, they are not done routinely after each course of antibiotic therapy. A visit to the physician's office cannot validate that all the medication was taken. A follow-up visit helps to determine if the infection has completely cleared. If the infection is resolved with one course of antibiotics, another course will not be prescribed. NURSING PROCESS STEP: Evaluation CLIENT NEEDS CATEGORY: Physiological integrity CLIENT NEEDS SUBCATEGORY: Physiological adaptation ****************************** --> QUESTION NUMBER _ 302 _ about (MC) QUESTION: "When teaching the parents of a 1-year-old child who is scheduled for placement of tympanostomy tubes, which of the following would the nurse include as the purpose for these tubes?" CHOICES ( X ) a.) Allow distribution of antibiotic solution into the middle ear. ( X ) b.) Assist in shrinking the mucosal lining of the middle ear. ( X ) c.) Increase the pressure in the middle ear. ( O ) d.) Allow ventilation and drainage of the middle ear. RATIONALE: Tympanostomy tubes allow ventilation of the middle ear and facilitate fluid drainage by maintaining the patency of the eustachian tube. The tubes help to equalize pressure but are not used to distribute medication into the ear. Decongestants, not tympanostomy tubes, may be used to shrink mucous membranes and improve eustachian tube function. Tympanostomy tubes help to equalize, not increase, the pressure in the middle ear. NURSING PROCESS STEP: Implementation CLIENT NEEDS CATEGORY: Physiological integrity CLIENT NEEDS SUBCATEGORY: Physiological adaptation ****************************** --> QUESTION NUMBER _ 303 _ about (MC) QUESTION: "When developing the postoperative plan of care for a child who is scheduled to have a tympanostomy tubes

inserted into the right ear, which of the following interventions would the nurse identify to facilitate drainage?" CHOICES ( X ) a.) Applying warm compresses to the right ear. ( O ) b.) Positioning the child to lie on the right side. ( X ) c.) Applying a gauze dressing to the left ear. ( X ) d.) Applying an ice pack to the left ear. RATIONALE: Positioning the child on the affected side, in this case the right side, will promote drainage from the middle ear by gravity. Application of heat, such as in the form of warm compresses, may facilitate drainage of exudate from the ear but only if the child is lying on the affected side. A gauze dressing is not applied after surgery. However, a loose wick may be inserted into the external ear canal to absorb drainage from the right ear. Application of an ice bag may help reduce pressure and edema. However, the ice bag would be applied to the right ear. NURSING PROCESS STEP: Planning CLIENT NEEDS CATEGORY: Physiological integrity CLIENT NEEDS SUBCATEGORY: Physiological adaptation ****************************** --> QUESTION NUMBER _ 304 _ about (MC) QUESTION: "Following insertion of tympanostomy tubes, offering liquid or soft foods to keep the child from chewing is written on the postoperative plan of care. Which of the following nursing diagnoses would this intervention address?" CHOICES ( X ) a.) Hyperthermia related to infectious process. ( X ) b.) Risk for Impaired Skin Integrity related to ear drainage. ( O ) c.) Pain related to the inflammatory process. ( X ) d.) Anxiety related to unfamiliar situation. RATIONALE: After tube insertion, the inflammatory process results in pain. Activities such as chewing often exacerbate this pain. Thus, the avoidance of chewing by offering liquid or soft foods is an intervention that addresses the nursing diagnosis of pain. For the nursing diagnosis of Hyperthermia, interventions typically would include removing bedclothes and extra clothing, reducing environmental temperature, and encouraging fluids. For the nursing diagnosis of Risk for Impaired Skin Integrity, appropriate measures would include keeping the skin around the ear clean and dry, cleansing with hydrogen peroxide, and protecting the skin with a protective coating. For the nursing diagnosis of Anxiety related to unfamiliar situation, measures would include teaching the child about procedures at a developmentally appropriate level, involving parents in care, and orienting the child to his surroundings. NURSING PROCESS STEP: Analysis, nursing diagnosis CLIENT NEEDS CATEGORY: Physiological integrity CLIENT NEEDS SUBCATEGORY: Physiological adaptation ****************************** --> QUESTION NUMBER _ 305 _ about (MC) QUESTION: "Which of the following would be the nurse's best response to the parents of a child being discharged from the day surgery center after insertion of tympanostomy tubes when they ask, "What will happen to the tubes in my child's ears?"" CHOICES

( X ) a.) "The tubes usually dissolve on their own in about 1 year." ( O ) b.) "You'll probably see them fall out in about 6 months." ( X ) c.) "The tubes must remain permanently in place." ( X ) d.) "Call for an appointment to have them removed in about 6 months." RATIONALE: The tympanostomy tubes, made of a polyurethane material that does not change in structure or composition while in the ear, usually remain in place for about 6 months then are spontaneously ejected from the ear. Parents should be told about the tubes' appearance so they can observe them if they fall out. The tubes do not change in structure or composition while in the ear. Therefore, they do not dissolve. The tubes typically are spontaneously ejected from the ear in about 6 months. The tubes do not need to be surgically removed because they typically are spontaneously ejected from the ear in about 6 months. NURSING PROCESS STEP: Implementation CLIENT NEEDS CATEGORY: Physiological integrity CLIENT NEEDS SUBCATEGORY: Physiological adaptation ****************************** --> QUESTION NUMBER _ 306 _ about (MC) QUESTION: "Which of the following nursing interventions would be most effective in helping a 2-year-old client stay quiet after a bronchoscopy?" CHOICES ( X ) a.) Allowing the child to go to the playroom. ( O ) b.) Having the parents stay at the bedside. ( X ) c.) Having the child play with another child in the room. ( X ) d.) Turning on the television so the child can watch cartoons. RATIONALE: A toddler has a short attention span and is energetic. Thus, keeping a 2-year-old child quiet is a challenge. Because the parents know their child well, the parents have a better chance of helping the child stay quiet. Therefore, they should be encouraged to stay with the child at the bedside. Allowing the child to go to the playroom would most likely encourage the child to be active rather than quiet. A 2-year-old child engages in parallel play but does not know how to play with others. A 2-year-old child's attention span is short, so watching television would keep the child quiet for only a short time. NURSING PROCESS STEP: Implementation CLIENT NEEDS CATEGORY: Physiological integrity CLIENT NEEDS SUBCATEGORY: Reduction of risk potential ****************************** --> QUESTION NUMBER _ 307 _ about (MC) QUESTION: "Which of the following suggestions would be most helpful to the parents of a 2-year-old child when managing the separation anxiety during hospitalization?" CHOICES ( X ) a.) Leave while the child is sleeping. ( O ) b.) Bring the child's favorite toys from home. ( X ) c.) Tell the child the time they are leaving and returning.

( X ) d.) Keep the visit time short. RATIONALE: Bringing a child's favorite toys, security blanket, or familiar objects from home can make the transition from home to hospital less stressful. The child receives comfort and reassurance from these items. Leaving without explaining may decrease the child's trust in the parents, ultimately adding to the child's level of anxiety. The parents should tell their toddler when they are leaving and when they will return, not by time but in relation to the child's usual activities (eg, by bedtime). Typically 2-year-old children have a limited sense of time. Short parental visits do not satisfy a toddler's overwhelming need for comfort because toddlers need to spend lots of time with parents due to separation anxiety. NURSING PROCESS STEP: Implementation CLIENT NEEDS CATEGORY: Psychosocial integrity CLIENT NEEDS SUBCATEGORY: None ****************************** --> QUESTION NUMBER _ 308 _ about (MC) QUESTION: "Which of the following assessments would be the priority for a 2-year-old child after a bronchoscopy?" CHOICES ( X ) a.) Cardiac rate. ( O ) b.) Respiratory quality. ( X ) c.) Sputum color. ( X ) d.) Pulse pressure changes.

RATIONALE: After bronchoscopy, airway obstruction secondary to laryngeal edema may occur. Therefore, assessment of the child's respiratory quality is the priority. The child should be observed for signs and symptoms of respiratory distress including tachypnea, increased stridor and retractions, and tachycardia. Assessing cardiac rate and rhythm is important and would be done once the client's respiratory status is assessed. Although observing the color of the sputum is an important assessment, it is not the priority. The sputum may be bloody after bronchoscopy. A change in pulse pressure is not associated with bronchoscopy but rather with intracranial pressure and shock. A pulse deficit is associated with some dysrhythmias. NURSING PROCESS STEP: Assessment CLIENT NEEDS CATEGORY: Physiological integrity CLIENT NEEDS SUBCATEGORY: Reduction of risk potential ****************************** --> QUESTION NUMBER _ 309 _ about (MC) QUESTION: "Which of the following would the nurse include when teaching the parents how to perform cardiopulmonary resuscitation for their 2-year-old child?" CHOICES ( X ) a.) Covering the child's nose and mouth with their own mouth when giving breaths ( X ) b.) Locating the brachial artery to check for the presence or absence of a pulse ( X ) c.) Placing the fingers for compression on an imaginary line between the child's nipples ( O ) d.) Giving one rescue breath for every five chest compressions RATIONALE: In a 2-year-old child, the ratio of breaths to compressions is 1:5. When giving breaths, the rescuer should pinch the nose shut. The mouth of the rescuer should make a seal over the child's mouth. The rescuer would make a seal over the nose and mouth of a child under the age of 1 year. In a child over the age of 1 year, the carotid pulse is palpated. Palpation of the brachial pulse is appropriate for the child under 1 year of age. Chest compressions on the child 1 to 8 years of age are applied to the lower sternum two finger breadths above the sternal notch. The imaginary

line between the child's nipples is the proper landmark for chest compressions in a child under 1 year of age. NURSING PROCESS STEP: Implementation CLIENT NEEDS CATEGORY: Physiological integrity CLIENT NEEDS SUBCATEGORY: Physiological adaptation ****************************** --> QUESTION NUMBER _ 310 _ about (MC) QUESTION: "A 10-year-old child with history of bronchial asthma triggered by exposure to cold, smoke, and nuts is brought to the hospital's emergency room by his mother. Appearing restless and anxious, the child has a respiratory rate of 36 breaths/minute and pulse rate of 160 bpm. Which of the following findings would be of greatest concern to the nurse?" CHOICES ( X ) a.) Increased respiratory effort. ( X ) b.) Moist, loose cough. ( O ) c.) Absence of wheezing. ( X ) d.) Prolonged expiratory phase. RATIONALE: Knowing that this child is most likely experiencing an asthmatic attack, the nurse would expect to hear wheezing and note some shortness of breath with a prolonged expiratory phase. However, of greatest concern would be the absence of wheezing indicating that the child is not moving air well through the lungs and is at risk for hypoxia and possible respiratory failure. Increased respiratory effort would be suspected secondary to bronchospasm associated with asthma. During an asthma attack, the cough usually is dry and sounds tight due to mucus accumulation and bronchoconstriction. Typically during an asthmatic attack, the client would demonstrate a prolonged expiratory phase because of air trapping and the increased effort to move air through constricted bronchioles. NURSING PROCESS STEP: Assessment CLIENT NEEDS CATEGORY: Physiological integrity CLIENT NEEDS SUBCATEGORY: Physiological adaptation ****************************** **--> QUESTION NUMBER _ 311 _ about (MC) QUESTION: "For the child diagnosed with an asthmatic attack, which of the following manifestations would best correlate with the child's arterial blood gas results, which include pH of 7.46, bicarbonate of 21, and a PCO 2 of 33 mm Hg?" CHOICES ( X ) a.) Greatly diminished breath sounds. ( O ) b.) A tingling sensation in the fingertips. ( X ) c.) Heart rate of 68 beats/minute. ( X ) d.) Absence of urination for several hours. RATIONALE: The arterial blood gas results indicate respiratory alkalosis. As the alkalinity of body fluids increases, ionization of calcium decreases. A low level of circulating ionized calcium increases the excitability of nerve and muscle tissue, manifested by paresthesia (numbness and tingling) of the digits, upper lip, and earlobes. In mild asthma with respiratory alkalosis, breath sounds are typically loud with expiratory wheezing. In mild asthma with respiratory alkalosis, the heart rate is usually elevated because of hyperventilation. In mild asthma with respiratory alkalosis, urine production is increased because of the increased renal circulation. As a result, bicarbonate, sodium,

and potassium excretion increases in an attempt to conserve hydrogen to correct the alkalosis. NURSING PROCESS STEP: Analysis CLIENT NEEDS CATEGORY: Physiological integrity CLIENT NEEDS SUBCATEGORY: Reduction of risk potential ****************************** --> QUESTION NUMBER _ 312 _ about (MC) QUESTION: "After staying several hours with her 10-year-old daughter who is admitted to the hospital with an asthmatic attack, the mother leaves to attend to her other children. The child exhibits continued signs and symptoms of respiratory distress. Which of the following findings would lead the nurse to make a nursing diagnosis of Anxiety related to respiratory distress?" CHOICES ( X ) a.) Complaints of an inability to get comfortable. ( O ) b.) Frequent requests for someone to stay in the room. ( X ) c.) Inability to remember her exact address. ( X ) d.) Verbalization of a feeling of tightness in her chest. RATIONALE: A 10-year-old child should be able to tolerate being alone. Frequently asking for someone to be in the room indicates a degree of psychological distress at this age suggesting Anxiety. The inability to get comfortable is commonly characteristic of child with a diagnosis of Pain. Inability to answer questions correctly may reflect a state of anoxia or a lack of knowledge. Tightness in the chest occurs as a result of bronchial spasms and indicates a diagnosis of Ineffective Airway Clearance. NURSING PROCESS STEP: Analysis CLIENT NEEDS CATEGORY: Physiological integrity CLIENT NEEDS SUBCATEGORY: Physiological adaptation ****************************** --> QUESTION NUMBER _ 313 _ about (MC) Dosage QUESTION: "Methylprednisolone (Solu-Medrol) 1 mg/kg is ordered for a child weighing 73 1/4 pounds. The medication is mixed so that 1 mL delivers 40 mg. How many milliliters of the medication should the nurse give?" CHOICES ( X ) a.) 0.18 mL. ( O ) b.) 0.83 mL. ( X ) c.) 1.83 mL. ( X ) d.) 3.85 mL. RATIONALE: Because the child weighs 33.3 kg (73 1/4 pounds divided by 2.2 equals 33.29 kg), set up the problem as follows: 1 mg/kg = 1x33.3 = 33.3 mg 40 mg/33.3 mg = 1 mL/x mL 40 mgxx mL = 33.3 mgx1 mL x mL = 33.3/40 Because the child weighs 33.3 kg (73 1/4 pounds divided by 2.2 equals 33.29 kg), set up the problem as follows: 1 mg/kg = 1x33.3 = 33.3 mg 40 mg/33.3 mg = 1 mL/x mL 40 mgxx mL = 33.3 mgx1 mL x mL = 33.3/40 Arriving at this answer means that the child's weight was not changed to kilograms. Because the child weighs 33.3 kg (73 1/4 pounds divided by 2.2 equals 33.29 kg), set up the problem as follows: 1 mg/kg = 1x33.3 = 33.3 mg 40 mg/33.3 mg = 1 mL/x mL 40 mgxx mL = 33.3 mgx1 mL x mL = 33.3/40 Because the child weighs 33.3 kg (73 1/4 pounds divided by 2.2 equals 33.29 kg), set up the problem as follows: 1 mg/kg = 1x33.3 = 33.3 mg 40 mg/33.3 mg = 1 mL/x mL 40 mgxx mL = 33.3 mgx1 mL x mL = 33.3/40 NURSING PROCESS STEP: Implementation CLIENT NEEDS CATEGORY: Physiological integrity CLIENT NEEDS SUBCATEGORY: Pharmacological and parenteral therapies ******************************

--> QUESTION NUMBER _ 314 _ about (MC) prednisone QUESTION: "Which of the following instructions would the nurse give to the parents of an 8-year-old child with asthma who is being switched from parenteral steroid therapy to a daily dose of oral prednisone?" CHOICES ( X ) a.) Administer the dose before bedtime to minimize side effects. ( X ) b.) Give the medication according to the child's response. ( O ) c.) Have the child take the dose with meals to prevent gastric irritation. ( X ) d.) Make sure the pill is given intact to maintain the enteric coating. RATIONALE: Prednisone causes severe gastric upset. Therefore, it should be given with food. It is recommended that the daily dose be given in the morning before 9:00 AM. Given at this time, the medication will suppress adrenal cortex activity less, which may reduce the risk of HPA:axis suppression. The drug must be given as ordered and not titrated to response. If the drug has been given over a long period, abrupt cessation can cause serious side effects. Because the pills are not enteric-coated, they may be crushed and mixed with food if the child has difficulty swallowing them. NURSING PROCESS STEP: Implementation CLIENT NEEDS CATEGORY: Physiological integrity CLIENT NEEDS SUBCATEGORY: Pharmacological and parenteral therapies ****************************** --> QUESTION NUMBER _ 315 _ about (MC) QUESTION: "When assessing the child with asthma for allergic rhinitis, which of the following would the nurse expect to find?" CHOICES ( O ) a.) Nasal crease. ( X ) b.) Abdominal pain. ( X ) c.) Fever. ( X ) d.) Mouth breathing. RATIONALE: In the child with asthma and allergic rhinitis, the allergic reaction to inhaled particles generally causes frequent nose rubbing, subsequently leading to a nasal crease. The child also may exhibit allergic shiners, dark circles under the eyes caused by nasal congestion. Typically abdominal pain, although associated with numerous disorders, is not related to allergic rhinitis. Fever, although a common assessment finding with numerous disorders, is unrelated to allergic rhinitis. Fever would be present if the child developed a subsequent infection secondary to the allergic rhinitis. Mouth breathing usually occurs when the child has enlarged tonsils or adenoids. NURSING PROCESS STEP: Assessment CLIENT NEEDS CATEGORY: Physiological integrity CLIENT NEEDS SUBCATEGORY: Physiological adaptation ****************************** --> QUESTION NUMBER _ 316 _ about (MC) peak expiratory flow meter QUESTION: "Which of the following statements by the parents of a child with asthma being taught the reasons for using a peak expiratory flow meter indicates the need for additional teaching?"

CHOICES ( O ) a.) "If there is no increase in flow after he gets his bronchodilator, we should give another treatment." ( X ) b.) "Finally, we have a way to monitor his condition and predict when he is getting worse." ( X ) c.) "This meter will help to monitor our child's condition, so changes can be made in therapy." ( X ) d.) "The meter readings will help us determine if he has other possible triggers besides dust and cold." RATIONALE: Although the meter does assist in evaluating the effectiveness of a treatment, repeating the dose of bronchodilator therapy is not recommended unless prescribed by a physician. Bronchodilators have serious side effects, and the child would need to be monitored closely if several treatments were given in a row. The peak expiratory flow meter is used to follow trends for diurnal variations that predict instability of asthma and need for increased therapy. It also assists in early detection of exacerbation of the asthma because decreases in the peak expiratory flow rate may indicate a worsening condition. The peak expiratory flow meter is used to monitor the asthma and assist in making decisions about increasing or decreasing therapy. By monitoring trends in readings and the child's condition, a peak expiratory flow meter can also be used to identify triggers of asthma. NURSING PROCESS STEP: Evaluation CLIENT NEEDS CATEGORY: Physiological integrity CLIENT NEEDS SUBCATEGORY: Reduction of risk potential ****************************** --> QUESTION NUMBER _ 317 _ about (MC) cromolyn sodium (Intal) QUESTION: "When discussing the use of cromolyn sodium (Intal) with the parent of a child diagnosed with asthma, the nurse should teach the mother that the medication will be ineffective if it is administered at which of the following?" CHOICES ( X ) a.) Intermittently for short-term use. ( O ) b.) During an asthmatic attack. ( X ) c.) Preparation for going to bed. ( X ) d.) Prior to riding a bicycle for a block. RATIONALE: Cromolyn sodium (Intal) is used as a prophylactic agent to help prevent bronchial asthmatic attacks. The drug inhibits histamine release and acts locally to prevent the release of mediator substances from mast (connective tissue) cells after exposure to allergens. The drug is not an anti-inflammatory, bronchodilator, or antihistamine agent. Therefore, it is of no use during an asthma attack. To be effective, cromolyn should be administered consistently over a long period of time. Short-term dosing provides no benefits. Cromolyn is used prophylactically and administered routinely several times a day. Although preparation for bed would not affect the effectiveness of cromolyn, it may be one of the scheduled dosing times. Although cromolyn is indicated for the prevention of exercise-induced bronchospasm, riding a bicycle one block is usually not considered to be strenuous exercise. Thus, cromolyn would not be helpful in preventing airway narrowing. NURSING PROCESS STEP: Implementation CLIENT NEEDS CATEGORY: Physiological integrity CLIENT NEEDS SUBCATEGORY: Pharmacological and parenteral therapies ****************************** --> QUESTION NUMBER _ 318 _ about (MC) bronchopneumonia QUESTION: "A 3-year-old child with cystic fibrosis is admitted to the hospital with bronchopneumonia. Which of the following signs and symptoms would be most helpful in providing supportive diagnostic data for this child's condition?" CHOICES

( X ) a.) Weight loss and stringy stools. ( O ) b.) Cough and fever. ( X ) c.) Constipation and vomiting. ( X ) d.) Dysuria and rash. RATIONALE: As a result of the infectious process and mucus accumulation, classic signs of pneumonia include fever and cough. Weight loss may occur in a child with cystic fibrosis because of the energy expenditure needed to fight the infection. Typically stools are large, bulky, and greasy. Constipation is not a common manifestation of pneumonia. However, vomiting may occur, especially if the child is coughing frequently and has a lot of mucus. Dysuria and rash are not associated with pneumonia. NURSING PROCESS STEP: Assessment CLIENT NEEDS CATEGORY: Physiological integrity CLIENT NEEDS SUBCATEGORY: Physiological adaptation ****************************** --> QUESTION NUMBER _ 319 _ about (MC) QUESTION: "On discharge, a child is ordered to receive ampicillin (Omnipen) four times a day. Which of the following instructions to the mother would be most appropriate?" CHOICES ( X ) a.) "Administer the drug at 9 AM, 1 PM, 5 PM, and 9 PM." ( X ) b.) "Give the drug during mealtime and with an evening snack." ( X ) c.) "Make sure that you give each dose about 4 hours apart." ( O ) d.) "Administer the drug every 6 hours around the clock." RATIONALE: It is important to give some antibiotics, like ampicillin, spaced at equal intervals over a 24-hour period to help maintain therapeutic levels of the medication in the bloodstream. If a medication is to be given four times in each 24-hour period, it should be given every 6 hours. Consulting with the caregiver about the best times within the prescribed schedule to give the medication gives the mother some control, thus promoting greater compliance. Ampicillin should be given every 6 hours to maintain therapeutic blood levels. Using a 9-1-5-9 schedule allows only 4 hours between 3 doses and 12 hours between the last dose. Thus, drug blood levels would be inconsistent impairing the drug's effectiveness. Giving the drug at mealtimes and with an evening snack is inappropriate because the time frame between doses would be inconsistent, leading to erratic drug blood levels. It is important to give some antibiotics, like ampicillin, spaced at equal intervals over a 24-hour period to help maintain therapeutic levels of the medication in the bloodstream. If a medication is to be given four times in each 24-hour period, it should be given every 6 hours. NURSING PROCESS STEP: Implementation CLIENT NEEDS CATEGORY: Physiological integrity CLIENT NEEDS SUBCATEGORY: Pharmacological and parenteral therapies ****************************** --> QUESTION NUMBER _ 320 _ about (MC) CF QUESTION: "Parents of a child with cystic fibrosis demonstrate knowledge of the effects of hot weather on their child when they state that hot weather is hazardous because the child has which of the following?" CHOICES ( X ) a.) Poor ability to concentrate urine.

( X ) b.) Little skin pigment to prevent sunburn. ( X ) c.) Poorly functioning temperature control center. ( O ) d.) Abnormally high salt loss through perspiration. RATIONALE: One characteristic of cystic fibrosis is the excessive loss of salt through perspiration. Salt supplements are almost always necessary during warm weather or any other time the child with cystic fibrosis perspires more than usual. In the child with cystic fibrosis, the functioning of the sweat glands is the problem, causing abnormal amounts of salt to be lost with perspiration. The ability to concentrate urine is not the problem. Little skin pigment is not a condition associated with cystic fibrosis. A poorly functioning temperature control center is not a condition related to cystic fibrosis. NURSING PROCESS STEP: Evaluation CLIENT NEEDS CATEGORY: Physiological integrity CLIENT NEEDS SUBCATEGORY: Physiological adaptation ****************************** --> QUESTION NUMBER _ 321 _ about (MC) QUESTION: "Which of the following observations by the home care nurse indicates that the mother of a child receiving home intravenous nafcillin therapy requires further teaching?" CHOICES ( X ) a.) Allows the antibiotic to run into the child's vein over a period of 30 minutes. ( O ) b.) Flushes the venous access site with heparin 20 minutes after giving the antibiotic. ( X ) c.) Stops the infusion when the area around the insertion site becomes hard and reddened. ( X ) d.) Calls the home care nurse because the antibiotic solution will not infuse. RATIONALE: When administering intravenous antibiotics, heparin or saline should be used to flush the intravenous line as soon as the infusion is completed so that the line remains patent. Waiting for 20 minutes is too long. Although nafcillin can be given as a slow intravenous push, it is usually infused over 30 minutes to decrease inflammation of the vein. The infusion should be stopped if there is any question about whether or not the fluid is entering a vein or subcutaneous tissue, evidenced by hardening or reddening of the site. If the intravenous access is not allowing infusion of the medication, the nurse should be called. NURSING PROCESS STEP: Evaluation CLIENT NEEDS CATEGORY: Physiological integrity CLIENT NEEDS SUBCATEGORY: Pharmacological and parenteral therapies ****************************** --> QUESTION NUMBER _ 322 _ about (MC) SIDS QUESTION: "The parents of a 3-month-old infant have been told that their infant has died of sudden infant death syndrome. Which of the following interventions is most important to include in the plan of care to assist the parents with their grieving process?" CHOICES ( X ) a.) Reassuring them that the infant's death was not their fault. ( O ) b.) Providing an opportunity for them to see the infant. ( X ) c.) Asking them if they would like to call their religious advisor. ( X ) d.) Giving them a package containing the infant's clothing.

10

RATIONALE: The parents should be given the opportunity to say their final farewells to their infant. This last contact helps them focus on the reality of the infant's death. Although helpful, reassuring them that they are not at fault does not focus on the reality of death, which is most important at this time. The presence of their pastor may be helpful but enabling them to see their child would be more important. For some parents, clothes may be too painful a reminder of their child's death, and they may not wish to take them home. NURSING PROCESS STEP: Implementation CLIENT NEEDS CATEGORY: Psychosocial integrity CLIENT NEEDS SUBCATEGORY: None ****************************** --> QUESTION NUMBER _ 323 _ about (MC) QUESTION: "Which of the following concepts would the nurse incorporate into the plan of care for a 4-year-old child to psychologically prepare the child for cardiac catheterization?" CHOICES ( X ) a.) Anxiety decreases when a preschooler is protected from learning about unpleasant events. ( X ) b.) Preschoolers are unable to understand the procedure. ( X ) c.) Little psychological preparation can be given to preschoolers. ( O ) d.) Preparation is a joint responsibility of the physician, parents, and nurse. RATIONALE: For a preschooler, psychological preparation for events is the joint responsibility of the physician, parents, and nurse, each playing a major role in caring for the child and meeting specific needs. Overprotecting a preschooler from unpleasant events can increase anxiety rather than decrease it, because the child needs to learn how to cope with stress. Preschoolers are ready to understand information that is individualized to their level. Little psychological preparation can be given to infants and toddlers. NURSING PROCESS STEP: Planning CLIENT NEEDS CATEGORY: Psychosocial integrity CLIENT NEEDS SUBCATEGORY: None ****************************** --> QUESTION NUMBER _ 324 _ about (MC) QUESTION: "Which of the following assessments would be the highest priority after a cardiac catheterization of a 5year-old child with a pressure dressing on the right extremity and an intravenous line in place who is slightly drowsy?" CHOICES ( X ) a.) Assessing for a gag reflex. ( X ) b.) Checking for pulses above the catheterization site. ( X ) c.) Checking the temperature of the left leg. ( O ) d.) Comparing color in the right and left legs. RATIONALE: Comparing the involved and uninvolved extremities in terms of color, temperature, pedal pulses, and capillary filling time is the highest priority following a cardiac catheterization to ensure adequate circulation to the involved extremity. Vital signs, including blood pressure, are checked as often as every 15 minutes after the procedure to detect dysrhythmias and hypotension. The priority following a cardiac catheterization is assessing the circulatory status. Because the child received local anesthesia, the gag reflex would be normal. However, fluids should be encouraged after the procedure because the dye used during the catheterization procedure causes osmotic diuresis. Pulses, especially those below the catheterization site, are checked for equality and symmetry. Checking the temperature is only one aspect of assessing the circulatory status of the extremities. Additionally, the involved and

11

uninvolved extremities must be compared to identify any differences indicating a compromise in circulation of the involved extremity. NURSING PROCESS STEP: Assessment CLIENT NEEDS CATEGORY: Physiological integrity CLIENT NEEDS SUBCATEGORY: Reduction of risk potential ****************************** --> QUESTION NUMBER _ 325 _ about (MC) QUESTION: "On admission for heart surgery to repair tetralogy of Fallot, the nurse observes that a 6-year-old child is cyanotic. The nurse judges that the parents understand this disorder when they explain that one of the underlying causes of their child's cyanosis is related to which of the following?" CHOICES ( X ) a.) Constriction of the aorta. ( X ) b.) Stenosis of the mitral valve. ( O ) c.) Stenosis of the pulmonary artery. ( X ) d.) Aorta receiving blood directly from the vena cava. RATIONALE: The three congenital defects associated with tetralogy of Fallot are (1) stenosis of the pulmonary artery, (2) interventricular septal defect, and (3) deviation of the aorta. A possible fourth defect is hypertrophy of the right ventricle that occurs as an adaptive mechanism to help overcome the effects of the three congenital defects. When pulmonary stenosis is severe, the child is cyanotic because insufficient blood reaches the lungs for good oxygenation. The child with tetralogy of Fallot experiences a deviation of the aorta, not constriction. The three congenital defects associated with tetralogy of Fallot are (1) stenosis of the pulmonary artery, (2) interventricular septal defect, and (3) deviation of the aorta. A possible fourth defect is hypertrophy of the right ventricle that occurs as an adaptive mechanism to help overcome the effects of the three congenital defects. The child with tetralogy of Fallot experiences a stenosis of the pulmonary artery not the mitral valve. The three congenital defects associated with tetralogy of Fallot are (1) stenosis of the pulmonary artery, (2) interventricular septal defect, and (3) deviation of the aorta. A possible fourth defect is hypertrophy of the right ventricle that occurs as an adaptive mechanism to help overcome the effects of the three congenital defects. The three congenital defects associated with tetralogy of Fallot are (1) stenosis of the pulmonary artery, (2) interventricular septal defect, and (3) deviation of the aorta. A possible fourth defect is hypertrophy of the right ventricle that occurs as an adaptive mechanism to help overcome the effects of the three congenital defects. NURSING PROCESS STEP: Evaluation CLIENT NEEDS CATEGORY: Physiological integrity CLIENT NEEDS SUBCATEGORY: Physiological adaptation ****************************** --> QUESTION NUMBER _ 326 _ about (MC) QUESTION: "When teaching the parents about an echocardiogram scheduled for their child, the nurse should explain that the primary reason for this procedure is to determine which of the following?" CHOICES ( O ) a.) Cardiac muscle structure. ( X ) b.) Pressure of the blood in the heart. ( X ) c.) Amount of blood entering the heart. ( X ) d.) Various sounds with each heartbeat. RATIONALE: An echocardiogram records the structure of the heart muscle and provides a graphic representation of

12

the heart working. An echocardiogram does not provide information about the pressure of the blood in the heart. Cardiac catheterization is used to measure the pressure in the heart chambers and major vessels and the amount of blood entering the heart. A cardiac catheterization, not echocardiogram, measures the amount of the blood entering the heart. Auscultation with a stethoscope is required to detect the various sounds made with each heartbeat. A phonocardiogram provides a graphic representation of heart sounds. NURSING PROCESS STEP: Implementation CLIENT NEEDS CATEGORY: Physiological integrity CLIENT NEEDS SUBCATEGORY: Reduction of risk potential ****************************** --> QUESTION NUMBER _ 327 _ about (MC) QUESTION: "Which of the following instructions would the nurse include when teaching the parents of a child who is to receive digoxin?" CHOICES ( X ) a.) Digoxin should be given with at least a full glass of water. ( O ) b.) Digoxin is absorbed better when taken 1 hour before eating. ( X ) c.) Signs of digoxin toxicity include increased heart rate and loss of appetite. ( X ) d.) The dose is repeated if the child vomits 30 minutes after taking the medication. RATIONALE: Absorption of digoxin is enhanced when the drug is taken on an empty stomach. Therefore, giving digoxin 1 hour before meals or 2 hours after meals results in better drug absorption. Digoxin does not need to be taken with large amounts of fluid. Digoxin toxicity typically results in a decreased heart rate. Loss of appetite is possible with digoxin toxicity. A digoxin dose is not repeated if the child vomits 30 minutes after ingestion because there would be no way to ascertain how much of the dose had been absorbed. Repeating the dose increases the child's risk for digoxin toxicity. NURSING PROCESS STEP: Implementation CLIENT NEEDS CATEGORY: Physiological integrity CLIENT NEEDS SUBCATEGORY: Pharmacological and parenteral therapies ****************************** --> QUESTION NUMBER _ 328 _ about (MC) QUESTION: "The mother of a child with tetralogy of Fallot asks the nurse why her child has clubbed fingers. The nurse bases the response on the understanding that clubbing is due to which of the following?" CHOICES ( X ) a.) Anemia. ( O ) b.) Peripheral hypoxia. ( X ) c.) Delayed physical growth. ( X ) d.) Destruction of bone marrow. RATIONALE: Clubbing of the fingers is one common finding in the child with persistent hypoxia leading to tissue changes in the body because of the low oxygen content of the blood (hypoxemia). It apparently results from tissue fibrosis and hypertrophy from the hypoxemia and from an increase in capillaries in the area, which occur as the body attempts to improve blood supply. Clubbing of the fingers is associated with polycythemia, not anemia. Polycythemia results from the body's attempt to increase oxygen levels in the tissues. The child may be small for his or her chronological age, but clubbing does not result from slow physical growth. Destruction of the bone marrow is not related to this congenital heart malformation. Instead, bone marrow is actively producing erythrocytes to compensate for the chronic hypoxia. NURSING PROCESS STEP: Implementation CLIENT NEEDS CATEGORY: Physiological integrity

13

CLIENT NEEDS SUBCATEGORY: Physiological adaptation ****************************** --> QUESTION NUMBER _ 329 _ about (MC) QUESTION: "When assessing a child with suspected mental retardation, which of the following behaviors would the nurse expect as least characteristic of a delay in early development?" CHOICES ( X ) a.) Lack of use of expressive language. ( X ) b.) Poor response to verbal commands. ( X ) c.) Onset of walking at age 20 months. ( O ) d.) Sitting up ability at age 6 months. RATIONALE: Being able to sit up at age 6 months is a typical developmental skill of a normal infant that could be expected to be delayed in a mentally retarded infant. Mentally retarded children tend to not use expressive language. Children with mental retardation commonly do not respond to verbal commands at a level appropriate to their chronological age. Walking, which normally occurs at about age 1 year, is almost always delayed in mentally retarded children. NURSING PROCESS STEP: Assessment CLIENT NEEDS CATEGORY: Physiological integrity CLIENT NEEDS SUBCATEGORY: Physiological adaptation ****************************** --> QUESTION NUMBER _ 330 _ about (MC) QUESTION: "A child with Down syndrome has an intelligence quotient (IQ) of about 40. The nurse would expect which of the following as the type of environment and interdisciplinary program to most likely benefit this child?" CHOICES ( X ) a.) Custodial. ( X ) b.) Institutional. ( O ) c.) Task analysis. ( X ) d.) Vocational training. RATIONALE: With advances in care of the mentally retarded, habit training task analysis, a step-by-step process in which each step is taught before moving onto the next step, would be most beneficial. Studies have shown that most people with IQs between about 35 and 50 can learn to take care of their hygienic needs, use acceptable social manners, and manage speech and other simple means of communication. Custodial care is required for the severely and profoundly retarded, those with IQs below about 35. The decision for institutionalization is dependent on multiple factors, only one of which is IQ. Vocational training would be appropriate for those with mild retardation with an IQ between 50 to 70. NURSING PROCESS STEP: Planning CLIENT NEEDS CATEGORY: Physiological integrity CLIENT NEEDS SUBCATEGORY: Physiological adaptation ****************************** --> QUESTION NUMBER _ 331 _ about (MC) QUESTION: "When discussing plans for genetic counseling with the parents of a child with Down syndrome, which of the

14

following would the nurse include as the primary role of the genetic team when working with a family?" CHOICES ( O ) a.) Providing parents with information about the risks of birth defects. ( X ) b.) Reporting the findings of chromosome analysis of the amniotic cells. ( X ) c.) Preparing the parents psychologically for the birth of a defective child. ( X ) d.) Prescribing birth control or abortion measures for the parents as needed. RATIONALE: The primary aim of genetic counseling is to inform clients of birth defect risks and the disorder to help the family understand and adjust to the disorder. Reporting results of chromosome analysis of amniotic cells is a secondary role of the team. Preparing a couple psychologically for the birth of a defective child is a secondary role. Although suggestions may be offered, the decision about birth control methods should be left to the couple. NURSING PROCESS STEP: Implementation CLIENT NEEDS CATEGORY: Safe, effective care environment CLIENT NEEDS SUBCATEGORY: Management of care ****************************** --> QUESTION NUMBER _ 332 _ about (MC) RF QUESTION: "When obtaining a health history from the mother of a 7-year-old child diagnosed with acute rheumatic fever, the nurse would focus questions to determine if the child was recently ill with which of the following?" CHOICES ( X ) a.) Measles. ( X ) b.) Mumps. ( O ) c.) Sore throat. ( X ) d.) Influenza virus. RATIONALE: Rheumatic fever is an inflammatory collagen disease that typically follows an infection by group A betahemolytic streptococci, ordinarily occurring in the throat. Rheumatic fever generally follows infection with streptococci within about 2 weeks. It is believed that the disease involves an autoimmune or allergic response to the organism. Rheumatic fever typically follows an infection by group A beta-hemolytic streptococci, ordinarily occurring in the throat. Infection with measles, a virus, does not predispose the child to develop rheumatic fever. Rheumatic fever typically follows an infection by group A beta-hemolytic streptococci, ordinarily occurring in the throat. Infection with mumps, a virus, does not predispose the child to develop rheumatic fever. Rheumatic fever typically follows an infection by group A beta-hemolytic streptococci, ordinarily occurring in the throat. Infection with influenza virus does not predispose the child to develop rheumatic fever. NURSING PROCESS STEP: Assessment CLIENT NEEDS CATEGORY: Physiological integrity CLIENT NEEDS SUBCATEGORY: Physiological adaptation ****************************** --> QUESTION NUMBER _ 333 _ about (MC) RF QUESTION: "On initial assessment of a 7-year-old child with rheumatic fever, which of the following would require contacting the physician immediately?" CHOICES ( O ) a.) Heart rate of 150 bpm.

15

( X ) b.) Swollen and painful knee joints. ( X ) c.) Twitching in the extremities. ( X ) d.) Red rash on the trunk. RATIONALE: A heart rate of 150 bpm is very high for a 7-year-old child and may indicate carditis. For this age group, the normal heart rate while awake is 70 to 110 bpm. Swollen and painful joints, such as the knee, are characteristic findings in the child with rheumatic fever and do not require immediate physician notification. Twitching in the extremities, known as chorea, is a characteristic finding in a child with rheumatic fever and does not require immediate physician notification. A red rash on the trunk typically indicates rheumatic fever and does not require immediate physician notification. NURSING PROCESS STEP: Analysis CLIENT NEEDS CATEGORY: Health promotion and maintenance CLIENT NEEDS SUBCATEGORY: None ****************************** --> QUESTION NUMBER _ 334 _ about (MC) RF QUESTION: "Which of the following statements indicates that the parents understand the need for their child to receive long-term antibiotic therapy after an episode of rheumatic fever?" CHOICES ( O ) a.) "It will prevent further streptococcal infections." ( X ) b.) "It will protect against further joint damage." ( X ) c.) "The inflammation will subside more quickly." ( X ) d.) "The inflammation will be reduced with future attacks." RATIONALE: Long-term treatment for rheumatic fever involves monthly penicillin injections to prevent subsequent streptococcal infections that can cause further heart damage. Rheumatic fever does not cause joint damage. There is no indication that inflammation subsides with fewer side effects while the child receives long-term antibiotic therapy. The inflammation subsides with bed rest and aspirin or steroids. Use of long-term antibiotic therapy has no relationship to future attacks and inflammation. NURSING PROCESS STEP: Evaluation CLIENT NEEDS CATEGORY: Physiological integrity CLIENT NEEDS SUBCATEGORY: Pharmacological and parenteral therapies ****************************** --> QUESTION NUMBER _ 335 _ about (MC) QUESTION: "The physician orders digoxin (Lanoxin) 0.15 mg PO daily for a child. The pharmacy supplies the digoxin in liquid form at a concentration of 0.05 mg/mL. How much of the medication should the nurse administer at each dose?" CHOICES ( X ) a.) 0.2 mL. ( X ) b.) 0.5 mL. ( O ) c.) 3.0 mL. ( X ) d.) 5.0 mL.

16

RATIONALE: The following calculation shows how to determine the correct amount of medication: 0.15 mg/x mL = 0.05 mg/1 mL 0.05x = 0.15 x = 0.15/0.05 x = 3 mL Thus, the correct dosage will be contained in 3 mL of the drug in solution. The following calculation shows how to determine the correct amount of medication: 0.15 mg/x mL = 0.05 mg/1 mL 0.05x = 0.15 x = 0.15/0.05 x = 3 mL Thus, the correct dosage will be contained in 3 mL of the drug in solution. The following calculation shows how to determine the correct amount of medication: 0.15 mg/x mL = 0.05 mg/1 mL 0.05x = 0.15 x = 0.15/0.05 x = 3 mL Thus, the correct dosage will be contained in 3 mL of the drug in solution. The following calculation shows how to determine the correct amount of medication: 0.15 mg/x mL = 0.05 mg/1 mL 0.05x = 0.15 x = 0.15/0.05 x = 3 mL Thus, the correct dosage will be contained in 3 mL of the drug in solution. NURSING PROCESS STEP: Implementation CLIENT NEEDS CATEGORY: Physiological integrity CLIENT NEEDS SUBCATEGORY: Pharmacological and parenteral therapies ****************************** --> QUESTION NUMBER _ 336 _ about (MC) QUESTION: "A 7-year-old child is to receive 10 grains of acetylsalicylic acid (aspirin) every 4 hours. Which of the following amounts would the nurse administer?" CHOICES ( X ) a.) 0.065 g. ( O ) b.) 0.65 g. ( X ) c.) 6.5 g. ( X ) d.) 65 g. RATIONALE: Ten grains of a drug is equivalent to 0.65 g. Ten grains of a drug is equivalent to 0.65 g. One grain would be equivalent to 0.065 g. Ten grains of a drug is equivalent to 0.65 g. One-hundred grains would be equivalent to 6.5g. Ten grains of a drug is equivalent to 0.65 g or 65 mg. One thousand grains would be equivalent to 65 g. NURSING PROCESS STEP: Implementation CLIENT NEEDS CATEGORY: Physiological integrity CLIENT NEEDS SUBCATEGORY: Pharmacological and parenteral therapies ****************************** --> QUESTION NUMBER _ 337 _ about (MC) QUESTION: "Which of the following signs or symptoms would lead the nurse to suspect that a 10-year-old child is experiencing early salicylate toxicity?" CHOICES ( X ) a.) Chest pain. ( X ) b.) Pink-colored urine. ( X ) c.) Slowed pulse rate. ( O ) d.) Dizziness. RATIONALE: Signs and symptoms of early salicylate toxicity include tinnitus, disturbances in hearing and vision, and dizziness. Salicylate toxicity may cause nausea, vomiting, diarrhea, and bleeding from mucous membranes from longterm use. Chest pain, rarely occurring in children, is not associated with salicylate toxicity. Pink-colored urine is not associated with salicylate toxicity. A slowed pulse rate is not associated with salicylate toxicity. NURSING PROCESS STEP: Assessment CLIENT NEEDS CATEGORY: Physiological integrity CLIENT NEEDS SUBCATEGORY: Pharmacological and parenteral therapies

17

****************************** --> QUESTION NUMBER _ 338 _ about (MC) RF QUESTION: "Which of the following measures would the nurse institute to help minimize joint pain in a child with rheumatic fever?" CHOICES ( X ) a.) Massaging the affected joints. ( X ) b.) Applying ice to the affected joints. ( O ) c.) Limiting movement of the affected joints. ( X ) d.) Encouraging progressive weight bearing. RATIONALE: In rheumatic fever, the joints-especially the knees, ankles, elbows, and wrists-are painful, swollen, red, and hot to the touch. Limiting movement of the affected joints typically minimizes pain. Massaging the joints likely will not aid in pain relief because the pain is due to the disease process and subsequent inflammation in the joint. Applying ice to the affected joints likely will not aid in pain relief because of the inflammation, edema, and effusion in the joint tissue. Exercise should be avoided because of the increased workload placed on the heart muscle. This is in contrast to usual recommendations for clients with other forms of arthritis. Despite joint involvement in rheumatic fever, permanent deformities do not occur. NURSING PROCESS STEP: Implementation CLIENT NEEDS CATEGORY: Physiological integrity CLIENT NEEDS SUBCATEGORY: Basic care and comfort ****************************** --> QUESTION NUMBER _ 339 _ about (MC) RF ?? QUESTION: "The parents of a child with rheumatic fever express concern that their other children will develop the disease. Which of the following would be the nurse's best response?" CHOICES ( O ) a.) "This disease is usually not contagious." ( X ) b.) "Your other children are as likely to develop this disease." ( X ) c.) "Medicine is available to prevent this, so check with your doctor." ( X ) d.) "Your other children are all girls, so there is no need to worry." RATIONALE: Usually other children in the family do not get rheumatic fever. The disorder is not contagious. Usually other children in the family do not get rheumatic fever. The disorder is not contagious. There is no medicine to give the children as prophylactic therapy. They have already been exposed to their sibling's streptococcal infection. If the other children do not have a streptococcal infection at this time, they probably will not develop it now. Girls, just like boys, are at risk for developing rheumatic fever. NURSING PROCESS STEP: Implementation CLIENT NEEDS CATEGORY: Physiological integrity CLIENT NEEDS SUBCATEGORY: Physiological adaptation ****************************** --> QUESTION NUMBER _ 340 _ about (MC) Sickle Cell QUESTION: "A 1-year-old child, the second child to have sickle cell disease in a family of five children, is admitted to the hospital with sickle cell crisis. Which of the following treatments would the nurse most likely expect to include in

18

the child's plan of care?" CHOICES ( X ) a.) Parenteral iron therapy. ( X ) b.) Exchange transfusion. ( O ) c.) Intravenous fluid therapy. ( X ) d.) Fast-acting anticoagulant therapy. RATIONALE: During a sickle cell crisis, increasing the transport and availability of oxygen to the body's tissues is paramount. Administering a high volume of intravenous fluid and electrolytes to help compensate for the acidosis resulting from hypoxemia associated with sickle cell crisis is one way to accomplish this. Fluid administration also helps overcome dehydration, a possible predisposing factor common in clients with sickle cell crisis. Iron therapy is contraindicated for this condition. Exchange transfusions are used only in certain situations, such as severe hyperbilirubinemia. Small amounts of blood are removed from the infant and replaced with whole blood. This helps to correct the anemia and lower bilirubin levels. Although anticoagulants have been suggested, they are not included in the usual treatment of sickle cell crisis. NURSING PROCESS STEP: Planning CLIENT NEEDS CATEGORY: Physiological integrity CLIENT NEEDS SUBCATEGORY: Pharmacological and parenteral therapies ****************************** --> QUESTION NUMBER _ 341 _ about (MC) Sickle Cell QUESTION: "A young child with sickle cell anemia prefers a side-lying position with the knees sharply flexed. The nurse would assess for further evidence of which of the following?" CHOICES ( X ) a.) Nausea. ( X ) b.) Backache. ( O ) c.) Abdominal pain. ( X ) d.) Emotional regression. RATIONALE: The child's self-positioning on the side with the knees sharply flexed strongly suggests the possibility of abdominal pain. The child assumes this position to decrease the discomfort. Thus, the nurse should assess for further evidence of abdominal pain. Nausea usually causes a young child to refuse nourishment. A backache would most probably cause the young child to lie supine to relieve discomfort. Regression is common in acutely ill hospitalized children, but insufficient data are given to confirm regression to early infancy. NURSING PROCESS STEP: Assessment CLIENT NEEDS CATEGORY: Physiological integrity CLIENT NEEDS SUBCATEGORY: Physiological adaptation ****************************** --> QUESTION NUMBER _ 342 _ about (MC) Sickle Cell QUESTION: "Which of the following would the nurse include about the need to seek prompt health care in the discharge teaching for the parents of a child following a sickle cell crisis?" CHOICES ( X ) a.) Headaches and nausea.

19

( X ) b.) Fatigue and lassitude. ( X ) c.) Skin rash and itching. ( O ) d.) Sore throat and fever. RATIONALE: Children with sickle cell disease are prone to develop infections as a result of the necrosis of areas within the body and a generalized less-than-optimal health status. If the child with sickle cell anemia develops signs of infection, such as sore throat and fever, prompt evaluation is necessary because an infection can precipitate a crisis. Fatigue, lassitude, headaches, and nausea could be prodromal signs of infection. However, they also could be signs of other illnesses. Fatigue, lassitude, headaches, and nausea could be prodromal signs of infection. However, they also could be signs of other illnesses. Skin rash and itching usually do not indicate an infection, but rather a possible contact dermatitis. The exception would be varicella. NURSING PROCESS STEP: Implementation CLIENT NEEDS CATEGORY: Physiological integrity CLIENT NEEDS SUBCATEGORY: Reduction of risk potential ****************************** --> QUESTION NUMBER _ 343 _ about (MC) Sickle Cell QUESTION: "The nurse reviews with the parents how to care for their child with sickle cell anemia at home. The nurse determines that the parents understand the basic principles of care when they state that they will do which of the following?" CHOICES ( X ) a.) Keep the child with them at all times. ( X ) b.) Restrict the child's fluids at night. ( O ) c.) Encourage their child to drink as much liquid as possible. ( X ) d.) Not allow their child to play with other children. RATIONALE: Because sickle cells tend to "log jam" in capillaries, it is important that the child receive adequate fluids. The fluids increase the blood volume and help prevent the log jam action that occurs during a crisis. The parents need to allow the child some independence for normal development. Keeping the child with them at all times will lead to overprotectiveness and dependency. Children with sickle cell disease should drink plenty of fluids day and night to prevent the "log jam" of the sickle cells in the capillaries. Children with a chronic illness need to be around other children for normal growth and development. However, this child should not be around anyone with an active infection. NURSING PROCESS STEP: Evaluation CLIENT NEEDS CATEGORY: Physiological integrity CLIENT NEEDS SUBCATEGORY: Physiological adaptation ****************************** --> QUESTION NUMBER _ 344 _ about (MC) Sickle Cell QUESTION: "The parents of a child with sickle cell anemia ask about the chances of sickle cell disease occurring in future children. The nurse responds based on the knowledge that both parents are carriers. Therefore, the risk of one of their children having the disease would be which of the following?" CHOICES ( X ) a.) 1 chance in 4 of their children. ( O ) b.) 1 chance in 4 for each pregnancy. ( X ) c.) 1 chance in 2 of their children.

20

( X ) d.) 1 chance in 2 for each pregnancy. RATIONALE: Sickle cell disease is an autosomal recessive Mendelian disorder. Therefore, if both parents have the trait, there is a 1 in 4 chance that any child (each pregnancy) will have the disease. Sickle cell disease is an autosomal recessive Mendelian disorder. Therefore, if both parents have the trait, there is a 1 in 4 chance that any child (each pregnancy) will have the disease. Sickle cell disease is an autosomal recessive Mendelian disorder. Therefore, if both parents have the trait, there is a 1 in 4 chance that any child (each pregnancy) will have the disease and a 1 in 2 chance that a child (each pregnancy) will have the trait. Sickle cell disease is an autosomal recessive Mendelian disorder. Therefore, if both parents have the trait, there is a 1 in 4 chance that any child (each pregnancy) will have the disease and a 1 in 2 chance that a child (each pregnancy) will have the trait. NURSING PROCESS STEP: Implementation CLIENT NEEDS CATEGORY: Physiological integrity CLIENT NEEDS SUBCATEGORY: Physiological adaptation ****************************** --> QUESTION NUMBER _ 345 _ about (MC) QUESTION: "After teaching the mother of a toddler with iron deficiency anemia about diet modifications, the nurse determines that the teaching was initially effective when the mother verbalizes dietary changes involving which of the following?" CHOICES ( X ) a.) Ingestion of equal amounts of iron-rich solids and milk products. ( O ) b.) Increased intake of iron-rich solids and decreased milk intake. ( X ) c.) Provision of several meals per day to the child. ( X ) d.) Twice-daily offerings of dairy food snacks to the child. RATIONALE: In iron deficiency anemia, the child's intake of iron-rich solids needs to be increased, while the intake of milk, which is low in iron, needs to be decreased to 1 quart per day. Decreasing milk intake will increase the child's hunger for and tolerance of solids that contain higher amounts of iron. The intake of foods rich in iron must be increased while the consumption of milk must be decreased. It is impossible to obtain the needed iron from milk alone, but milk does contain essential minerals and vitamins. Providing the child with several meals per day does not ensure that the solid foods will be consumed. Offering the child snacks is appropriate, but the amount of iron-rich food consumed must be increased while the amount of milk intake must be decreased. Therefore, the snacks should be solid foods high in iron. NURSING PROCESS STEP: Evaluation CLIENT NEEDS CATEGORY: Physiological integrity CLIENT NEEDS SUBCATEGORY: Physiological adaptation ****************************** --> QUESTION NUMBER _ 346 _ about (MC) QUESTION: "When discussing the appropriate iron-rich food selections with the mother of an 11-month-old infant with iron deficiency anemia, which of the following choices verbalized by the mother indicates successful teaching?" CHOICES ( O ) a.) Eggs, fortified cereals, meats, and green vegetables. ( X ) b.) Fruits, cereals, milk, and yellow vegetables. ( X ) c.) Eggs, fruits, milk, and mixed vegetables.

21

( X ) d.) Juices, fruits, fortified cereals, and milk. RATIONALE: Relatively high amounts of iron are contained in eggs, fortified cereals, meats, and green vegetables. Although fruits, nonfortified cereals, milk, and yellow vegetables contain iron, they are not the best choices for ironrich foods. Although eggs are an appropriate choice for iron-rich foods, fruits, mixed vegetables, and milk contain less iron. Fortified cereals are excellent sources of iron; however, juices, fruits, and milk contain less iron and are not the best choices. NURSING PROCESS STEP: Evaluation CLIENT NEEDS CATEGORY: Physiological integrity CLIENT NEEDS SUBCATEGORY: Physiological adaptation ****************************** --> QUESTION NUMBER _ 347 _ about (MC) QUESTION: "Which of the following instructions would be most appropriate for the nurse to include in the teaching plan for the mother of a 1-year-old child who is to receive iron therapy with ferrous sulfate (Feosol) drops?" CHOICES ( X ) a.) Mix the drops with the child's mealtime cup of milk. ( O ) b.) Put the drops in the child's mouth, then follow with juice. ( X ) c.) Put the drops in the child's mouth, then follow with milk. ( X ) d.) Mix the drops in the child's bedtime water bottle. RATIONALE: Absorption of iron is enhanced in an acid environment. Thus, iron drops are better absorbed when mixed with fruit juice or followed by fruit juice. Milk tends to decrease iron absorption. Medications should not be mixed in a cup or bottle of fluids. If the child does not drink the entire cup or bottle, it is difficult to determine how much of the medication the child actually received. Also, even though the child may be learning to use a cup, it may spill, again making it difficult to determine the amount of medication that the child has received. Putting the drops in the child's mouth is appropriate. However, following administration with milk should be avoided because milk decreases iron absorption. Medications should not be mixed in a bottle of fluids. If the child does not drink the entire bottle, it is difficult to determine how much of the medication the child actually received. NURSING PROCESS STEP: Planning CLIENT NEEDS CATEGORY: Physiological integrity CLIENT NEEDS SUBCATEGORY: Pharmacological and parenteral therapies ****************************** --> QUESTION NUMBER _ 348 _ about (MC) QUESTION: "Which of the following information obtained during the nursing history would help support a client's diagnosis of hemophilia?" CHOICES ( X ) a.) A brother and sister who are healthy. ( X ) b.) Italian and German ethnic background. ( O ) c.) A maternal uncle with prolonged postoperative bleeding. ( X ) d.) Paternal grandmother's death from chronic lymphocytic leukemia. RATIONALE: Hemophilia A is a genetically transmitted X-linked recessive disorder characterized by a deficiency of plasma factor VIII. A hemophiliac man and a normal woman have normal male children and female children who carry

22

the hemophilia trait. The carrier females pass the abnormal gene to half their sons and the carrier trait to half their daughters. The mother's brother most likely has hemophilia, as evidenced by the prolonged postoperative bleeding. Thus, the mother may be a carrier. A healthy brother and sister would not suggest an underlying genetic disorder. Ethnic background is not related to the development of hemophilia. History of familial leukemia is unrelated to the development of hemophilia. NURSING PROCESS STEP: Assessment CLIENT NEEDS CATEGORY: Physiological integrity CLIENT NEEDS SUBCATEGORY: Physiological adaptation ****************************** --> QUESTION NUMBER _ 349 _ about (MC) Bleeding episode QUESTION: "The neonate with hemophilia A experiences bleeding in the elbow and is seen in the emergency department. Which of the following nursing interventions would be most appropriate to minimize bleeding in the affected area?" CHOICES ( X ) a.) Applying continuous pressure to the elbow. ( X ) b.) Allowing the affected arm to dangle at the client's side. ( X ) c.) Placing a warm moist pack on the elbow. ( O ) d.) Elevating the elbow above the level of the heart. RATIONALE: When a bleeding episode occurs, the affected area should be immobilized and elevated to slow blood flow to the area and promote hemostasis. Pressure should be applied to the area for 10 to 15 minutes to promote clot formation. Pressure should be applied to the area for approximately 10 to 15 minutes to promote clot formation. Continuous pressure is not necessary. Allowing the client's arm to dangle at the side is ineffective in slowing blood flow to the area. Rather, the affected area should be elevated above the level of the heart to slow the blood flow. Using a warm moist pack on the elbow promotes vasodilation and bleeding. Instead, cold packs should be used to promote vasoconstriction. NURSING PROCESS STEP: Implementation CLIENT NEEDS CATEGORY: Physiological integrity CLIENT NEEDS SUBCATEGORY: Reduction of risk potential ****************************** --> QUESTION NUMBER _ 350 _ about (MC) QUESTION: "While attending a support group, the parents of a child with hemophilia become concerned because several of the families have had older children who have died from acquired immunodeficiency syndrome (AIDS). They ask the nurse how these children got the AIDS virus. The nurse bases the response on which of the following as the most likely route of transmission of AIDS to these children?" CHOICES ( O ) a.) Contamination of the factor VIII replacement received during bleeding episodes. ( X ) b.) Casual contact with a child testing positive for human immunodeficiency virus (HIV). ( X ) c.) Use of a contaminated needle to obtain a blood sample for type and crossmatching. ( X ) d.) Exposure in the waiting room to children with AIDS attending the same hematology clinic. RATIONALE: The AIDS virus is spread by direct contact with blood or blood products and by sexual contact. Children with hemophilia are at risk for AIDS because of the factor VIII concentrate infusions they receive. These concentrates are derived from larger quantities of pooled plasma, exposing recipients to thousands of blood donors. However, current testing of blood and blood products has reduced the risk of AIDS virus transmission. There is no evidence that

23

casual contact between infected and uninfected people transmits the HIV virus. All venipunctures for blood specimens in hospitals and clinics are performed with sterile disposable needles. Because the needles are sterile, they cannot be a source of HIV transmission. Exposure to others in a waiting room is considered casual contact. There is no evidence that casual contact between infected and uninfected people transmits the HIV virus. NURSING PROCESS STEP: Implementation CLIENT NEEDS CATEGORY: Physiological integrity CLIENT NEEDS SUBCATEGORY: Physiological adaptation ****************************** --> QUESTION NUMBER _ 351 _ about (MC) Leukemia QUESTION: "A child with leukemia presents with petechiae; gums, lips, and nose that bleed easily; and bruising on various parts of her body. Which of the following laboratory test results would the nurse correlate with these findings?" CHOICES ( O ) a.) Platelet count of 80 x 10 3/mm 3. ( X ) b.) Serum calcium level of 5 mg/dL. ( X ) c.) Fibrinogen level of 75 mg/dL. ( X ) d.) Partial thromboplastin time (PTT) of 38 seconds. RATIONALE: In leukemia, megakaryocytes, from which platelets are derived, are decreased. Normal platelet counts range from 150 to 300 x 10 3/mm 3. A platelet count of 80 x 10 3/mm 3 is low, predisposing the child to bruising and bleeding easily. Although the serum calcium level is decreased, low serum calcium levels are not related to bleeding and bruising in a child with leukemia or any aspect of leukemia. Normal fibrinogen level ranges from 200 to 400 mg/dL. However, insufficient fibrinogen concentration is not related to bleeding and bruising in a child with leukemia or any aspect of leukemia. PTT, a measurement of clotting factors (except factor XIII), normally ranges from 25 to 40 seconds. The child's PTT is within the normal range. NURSING PROCESS STEP: Analysis CLIENT NEEDS CATEGORY: Physiological integrity CLIENT NEEDS SUBCATEGORY: Reduction of risk potential ****************************** --> QUESTION NUMBER _ 352 _ about (MC) QUESTION: "When caring for the child with leukemia who is at risk for bleeding, which of the following measures should be avoided?" CHOICES ( X ) a.) Use of stool softeners. ( X ) b.) Frequent position changes in bed. ( X ) c.) Visits with friends and siblings. ( O ) d.) Intramuscular injections. RATIONALE: For a child with leukemia at risk for bleeding, all treatments should be performed gently. However, all injections should be limited or avoided as much as possible to reduce the risk of tissue injury and subsequent bleeding and bruising. Stool softeners are used to facilitate bowel elimination by preventing the passage of hard stool that may damage the rectal mucosa and subsequently lead to bleeding. Frequent position changes in bed are to be encouraged to maintain skin integrity and minimize the risk of trauma to the skin, thereby reducing the possibility of bleeding and bruising secondary to this trauma. Visits with friends and siblings are important for adequate growth and development of the child. Visitation would be curtailed if the child were at risk for possible infection secondary to a

24

decreased neutrophil count. NURSING PROCESS STEP: Implementation CLIENT NEEDS CATEGORY: Physiological integrity CLIENT NEEDS SUBCATEGORY: Reduction of risk potential ****************************** --> QUESTION NUMBER _ 353 _ about (MC) QUESTION: "How many tablets would the nurse give each day to a child who is ordered to receive mercaptopurine (Purinethol) 75 mg per day orally when the pharmacy supplies mercaptopurine packaged in 50-mg tablets for oral administration?" CHOICES ( X ) a.) 1/2 of 1 tablet. ( O ) b.) 1 1/2 tablets. ( X ) c.) 2 tablets. ( X ) d.) 2 1/2 tablets. RATIONALE: The nurse determines the number of 50-mg tablets of a drug to give when the client is to receive 75 mg of the drug for each dosage by using ratios, as follows: 1 tablet/50 mg = x tablets/75 mg 50x = 75 x = 75/50 x = 1.5 tablets The nurse determines the number of 50-mg tablets of a drug to give when the client is to receive 75 mg of the drug for each dosage by using ratios, as follows: 1 tablet/50 mg = x tablets/75 mg 50x = 75 x = 75/50 x = 1.5 tablets The nurse determines the number of 50-mg tablets of a drug to give when the client is to receive 75 mg of the drug for each dosage by using ratios, as follows: 1 tablet/50 mg = x tablets/75 mg 50x = 75 x = 75/50 x = 1.5 tablets The nurse determines the number of 50-mg tablets of a drug to give when the client is to receive 75 mg of the drug for each dosage by using ratios, as follows: 1 tablet/50 mg = x tablets/75 mg 50x = 75 x = 75/50 x = 1.5 tablets NURSING PROCESS STEP: Implementation CLIENT NEEDS CATEGORY: Physiological integrity CLIENT NEEDS SUBCATEGORY: Pharmacological and parenteral therapies ****************************** --> QUESTION NUMBER _ 354 _ about (MC) QUESTION: "Which of the following would the nurse include in the plan of care for a child with leukemia who has an absolute neutrophil count of 400/mm 3?" CHOICES ( O ) a.) Restrict staff and visitors with active infections. ( X ) b.) Place the child in strict isolation. ( X ) c.) Consult with the physician to administer an antiemetic. ( X ) d.) Increase the child's oral fluid intake. RATIONALE: The child's neutrophil count is low (normal range is 3,000 to 5,000 cells/mm 3), predisposing the child to infection. If an infection occurs, the child will have difficulty combating it. Therefore, staff and visitors are restricted to those without an active infection. Typically neutropenic precautions, not strict isolation would be used to protect the child from exposure to infection. The hospitalized child would be placed in a private room with visitors and staff screened for illnesses. Temperature would be monitored every 4 hours. Low neutrophil counts do not increase the likelihood of vomiting; therefore, an antiemetic is not needed. Increasing the child's oral fluid intake may be necessary; however, doing so is unrelated to the child's neutrophil count. NURSING PROCESS STEP: Planning CLIENT NEEDS CATEGORY: Safe, effective care environment CLIENT NEEDS SUBCATEGORY: Safety and infection control

25

****************************** --> QUESTION NUMBER _ 355 _ about (MC) Leukemia QUESTION: "Which of the following would the nurse teach the mother of a child with leukemia who has an absolute neutrophil count of 900/mm 3?" CHOICES ( X ) a.) The child should wear gloves when in contact with others. ( O ) b.) The child should stay away from crowds of people. ( X ) c.) Anyone in direct contact with the child must wear a gown and mask. ( X ) d.) The child should eat raw fruits and vegetables. RATIONALE: The normal neutrophil count range is 3,000 to 5,000 cells/mm 3. An absolute neutrophil count of 900/mm 3 is low, placing the child at high risk for infection. Therefore, the nurse should instruct the mother to have the child avoid crowds because of the risk of exposure to infection. Additionally, siblings and others who have an active infection should stay away from the child. A mask, not gloves, may be helpful for protecting the child from contracting infections transmitted by the airborne route. The child's absolute neutrophil count is high enough that a mask, gown, and isolation are not necessary. Raw fruits and vegetables contain bacteria and fungi. If ingested by a child with a low neutrophil count, the risk for infection increases. Therefore, their ingestion should be avoided. NURSING PROCESS STEP: Implementation CLIENT NEEDS CATEGORY: Safe, effective care environment CLIENT NEEDS SUBCATEGORY: Safety and infection control ****************************** --> QUESTION NUMBER _ 356 _ about (MC) QUESTION: "When caring for terminally ill children and their families, which of the following is recommended as most important for the nurse to have?" CHOICES ( X ) a.) Experience with the death of a loved one. ( X ) b.) Development of a belief that accepts life after death. ( X ) c.) Participation in a course examining how best to deal with death and grieving. ( O ) d.) A working personal philosophy concerning life and death. RATIONALE: Nurses caring for terminally ill clients are better prepared to do so when they have worked out a personal philosophy of death. Although other experiences such as having lost a loved one to death, taking classes in caring for dying clients and grieving, and developing a personal belief in a supreme being and a life hereafter may be helpful in assisting the nurse in thinking about death, the nurse's own feelings about life and death are the most important. Although other experiences such as having lost a loved one to death, taking classes in caring for dying clients and grieving, and developing a personal belief in a supreme being and a life hereafter may be helpful in assisting the nurse in thinking about death, the nurse's own feelings about life and death are the most important. Although other experiences such as having lost a loved one to death, taking classes in caring for dying clients and grieving, and developing a personal belief in a supreme being and a life hereafter may be helpful in assisting the nurse in thinking about death, the nurse's own feelings about life and death are the most important. NURSING PROCESS STEP: Implementation CLIENT NEEDS CATEGORY: Psychosocial integrity CLIENT NEEDS SUBCATEGORY: None ******************************

26

--> QUESTION NUMBER _ 357 _ about (MC) QUESTION: "Which of the following would be most appropriate for the nurse to include in the plan of care for a dying child to meet the child's emotional needs during the last days of life?" CHOICES ( X ) a.) Restrict visitors to the parents to avoid overtaxing the child. ( O ) b.) Answer the child's questions about illness and death honestly. ( X ) c.) Focus on the child's physical needs to attempt to prevent sadness. ( X ) d.) Encourage the child to play quietly with a roommate to provide pleasure. RATIONALE: Most clients, even children, are aware when death appears imminent. The best policy is to answer the child's questions honestly, thus helping the child feel less isolated and alone. Actions such as restricting visitors, concentrating on efforts to make the child think of something other than death, and encouraging the child to replace thoughts of sadness with thoughts of pleasurable things are not recommended. They tend to increase the dying child's fear, isolation, and feelings of loss of control. Actions such as restricting visitors, concentrating on efforts to make the child think of something other than death, and encouraging the child to replace thoughts of sadness with thoughts of pleasurable things are not recommended. They tend to increase the dying child's fear, isolation, and feelings of loss of control. Actions such as restricting visitors, concentrating on efforts to make the child think of something other than death, and encouraging the child to replace thoughts of sadness with thoughts of pleasurable things are not recommended. They tend to increase the dying child's fear, isolation, and feelings of loss of control. NURSING PROCESS STEP: Planning CLIENT NEEDS CATEGORY: Psychosocial integrity CLIENT NEEDS SUBCATEGORY: None ****************************** --> QUESTION NUMBER _ 358 _ about (MC) cleft lip QUESTION: "The parents of a neonate with a cleft lip are shocked when they see their child for the first time. Which of the following nursing actions would the nurse include in the neonate's plan of care to help the parents accept their infant's anomaly?" CHOICES ( X ) a.) Encouraging the parents to visit more frequently. ( X ) b.) Reassuring them that surgery will correct the defect. ( O ) c.) Showing them pictures of babies before and after corrective surgery. ( X ) d.) Allowing them to complete their grieving process before seeing the infant again. RATIONALE: Preoperative and postoperative pictures of babies with cleft palates and lips provide clear and concrete images of what to expect after corrective surgery. Providing these pictures is specific to the parents' behavior because the parents reflect societal values that emphasize an infant's facial appearance and responsive expressiveness. Encouraging the parents to visit more often may make them believe they are currently not visiting enough and could cause unwarranted guilt. There is no evidence they will not visit frequently. Although reassuring the parents that the defect can be corrected may be helpful, showing the parents pictures of infants before and after surgical correction provides them with objective evidence of what to expect. Allowing the completion of the grieving process before another interaction between the infant and parents could result in a separation for months. NURSING PROCESS STEP: Planning CLIENT NEEDS CATEGORY: Psychosocial integrity CLIENT NEEDS SUBCATEGORY: None ******************************

27

--> QUESTION NUMBER _ 359 _ about (MC) cleft lip QUESTION: "After surgical repair of a cleft lip, the infant exhibits difficulty breathing. Which of the following measures would the nurse institute first?" CHOICES ( X ) a.) Raising the infant's head. ( X ) b.) Turning the infant onto the abdomen. ( X ) c.) Administering oxygen per mask. ( O ) d.) Exerting downward pressure on the infant's chin. RATIONALE: After the repair of a cleft lip, the infant must become accustomed to nasal breathing. If the infant is having difficulty breathing, it would be best to open the mouth by exerting downward pressure on the chin. In some instances, an airway is used postoperatively; when there is no airway in place, it is best to try pressure on the chin first. Raising the infant's head would not ease the respiratory effort. The mouth needs to be opened. Turning the infant onto the abdomen would not ease the respiratory effort. The mouth needs to be opened. Oxygen is not necessary if opening the airway is successful. In any case, using a mask over the child's face may aggravate the problem and could potentially damage the suture line. NURSING PROCESS STEP: Implementation CLIENT NEEDS CATEGORY: Physiological integrity CLIENT NEEDS SUBCATEGORY: Physiological adaptation ****************************** --> QUESTION NUMBER _ 360 _ about (MC) cleft lip QUESTION: "Which of the following methods would the nurse use to feed an infant after surgical repair of cleft lip?" CHOICES ( X ) a.) Gastric gavage. ( X ) b.) Intravenous fluids. ( O ) c.) Rubber-tipped medicine dropper. ( X ) d.) Bottle with a lamb's nipple. RATIONALE: Feeding methods should produce the least tension possible on the sutures to promote effective healing of the cleft lip repair. Therefore, a rubber-tipped medicine dropper has been found to be a satisfactory method for feeding an infant who has had surgical repair of a cleft lip. Ordinarily, gastric gavage is not used unless the infant develops respiratory problems. Intravenous fluids do not supply complete nutrition for the infant; therefore, they would not be used as a feeding method. A lamb's nipple may be successful for feeding a child with a cleft palate once the lip is healed; however, the action of making a seal around the nipple would put tension on the suture line. NURSING PROCESS STEP: Implementation CLIENT NEEDS CATEGORY: Physiological integrity CLIENT NEEDS SUBCATEGORY: Physiological adaptation ****************************** --> QUESTION NUMBER _ 361 _ about (MC) QUESTION: "In the initial assessment, which of the following would the nurse expect as a typical sign of esophageal atresia and tracheoesophageal fistula (TEF)?"

28

CHOICES ( O ) a.) Continuous drooling. ( X ) b.) Diaphragmatic breathing. ( X ) c.) Bloody emesis. ( X ) d.) Large amounts of frothy meconium. RATIONALE: Esophageal atresia and TEF may occur together or separately. Esophageal atresia prevents the passage of swallowed mucus and saliva into the stomach. After fluid has accumulated in the pouch, it flows from the mouth. The infant then drools continuously. The lack of swallowed amniotic fluid prevents the accumulation of normal meconium; lack of stool results. Diaphragmatic breathing is not associated with TEF. Bloody emesis may occur with upper gastrointestinal bleeding or irritation. Meconium stool is usually passed within the first 24 hours after birth. It is dark and pasty, not frothy. For the infant with esophageal atresia and TEF, no stool would be passed. NURSING PROCESS STEP: Assessment CLIENT NEEDS CATEGORY: Physiological integrity CLIENT NEEDS SUBCATEGORY: Physiological adaptation ****************************** --> QUESTION NUMBER _ 362 _ about (MC) QUESTION: "For which of the following reasons would the nurse determine that a newborn's initial feeding attempts result in behaviors indicative of tracheoesophageal fistula (TEF)?" CHOICES ( X ) a.) Sucking attempts were too poorly coordinated to be effective. ( X ) b.) Projectile vomiting occurred after drinking 4 ounces. ( O ) c.) Coughing, choking, and cyanosis occurred after several swallows of formula. ( X ) d.) Sleeping occurred after 10 mL of formula with inability to be stimulated to take more. RATIONALE: The newborn with TEF swallows normally, but the fluids quickly fill the blind pouch. The infant then coughs, chokes, and becomes cyanotic while the fluid returns through the nose and mouth. Poor rooting reflexes and sucking attempts are typical of infants with neurologic dysfunction or related to reflex depression secondary to medication given to the mother during labor. Projectile vomiting is typical of infants with neurologic dysfunctions. This reflex may also be depressed by medication given to the mother during labor. Falling asleep after taking little formula is characteristic of an infant who becomes exhausted with the exertion of feeding, often caused by a cardiac anomaly. NURSING PROCESS STEP: Evaluation CLIENT NEEDS CATEGORY: Physiological integrity CLIENT NEEDS SUBCATEGORY: Physiological adaptation ****************************** --> QUESTION NUMBER _ 363 _ about (MC) QUESTION: "Before corrective surgery, the newborn with tracheoesophageal fistula (TEF) is placed on his back in a crib with his head and shoulders elevated. The nurse positions the infant to accomplish which of the following?" CHOICES ( X ) a.) Reduce cardiac workload that has been increased by the anomaly.

29

( X ) b.) Alleviate the pressure of the distended abdominal contents on the diaphragm. ( O ) c.) Enhance pooling of secretions in the bottom of the upper esophageal pouch. ( X ) d.) Allow air to escape from the fistula into the trachea to reduce gastric distention. RATIONALE: When an infant with TEF is placed on the back with the head and shoulders elevated, gravity encourages the flow of secretions with pooling in the bottom of the upper pouch. More effective removal of secretions can be accomplished by positioning a catheter in this pool of secretions. Generally, the infant with TEF experiences no additional cardiac workload and little possibility of cardiac failure unless other anomalies complicate TEF. With each breathing cycle, some air is forced into the stomach, preventing upward passage of air into the trachea. Eventually, abdominal distention would result. Air is forced into the stomach through the fistula, preventing air from moving up into the trachea. NURSING PROCESS STEP: Implementation CLIENT NEEDS CATEGORY: Physiological integrity CLIENT NEEDS SUBCATEGORY: Physiological adaptation ****************************** --> QUESTION NUMBER _ 364 _ about (MC) QUESTION: "When preparing for the discharge of a newborn who has undergone corrective surgery for a tracheoesophageal fistula (TEF), the nurse teaches the parents about the need for long-term health care because their child has a high probability of developing which of the following? " CHOICES ( X ) a.) Speech problems. ( O ) b.) Esophageal stricture. ( X ) c.) Gastric ulcers. ( X ) d.) Recurrent mild diarrhea with dehydration. RATIONALE: After corrective surgery for repair of TEF, the risk for esophageal stricture is high because scar tissue forms at the site of the esophageal anastomosis, often requiring dilation at the anastomosis site during the first years of childhood in about half the children. Speech problems are likely if other abnormalities are present to produce them. However, the larynx and structures of speech are not affected by TEF. Although dysphagia and strictures may decrease food intake and poor weight gain may be noted, gastric ulcers are not associated with TEF repair. Recurrent mild diarrhea with dehydration typically does not develop from surgery to correct TEF. NURSING PROCESS STEP: Implementation CLIENT NEEDS CATEGORY: Physiological integrity CLIENT NEEDS SUBCATEGORY: Reduction of risk potential ****************************** --> QUESTION NUMBER _ 365 _ about (MC) QUESTION: "Which of the following would the nurse expect to find on further assessment of a newborn with an imperforate anal membrane?" CHOICES ( O ) a.) No meconium stool. ( X ) b.) Abdominal distention. ( X ) c.) Ribbon-like stools.

30

( X ) d.) Hydrocele. RATIONALE: The absence of meconium stool is consistent with a diagnosis of imperforate anus because stool cannot be passed. Abdominal distention, a later sign of imperforate anus, occurs from the accumulation of gas and feces in the bowel. Ribbon-like stools are associated with anal stenosis. Hydrocele is not associated with anorectal malformations. NURSING PROCESS STEP: Assessment CLIENT NEEDS CATEGORY: Physiological integrity CLIENT NEEDS SUBCATEGORY: Physiological adaptation ****************************** --> QUESTION NUMBER _ 366 _ about (MC) NB QUESTION: "The father of a newborn observes that the neonate's big toe dorsiflexes and the other toes fan when the nurse gently strokes the sole of the foot. The nurse should interpret this positive finding as which of the following?" CHOICES ( X ) a.) Stepping reflex. ( X ) b.) Plantar grasp. ( X ) c.) Galant reflex. ( O ) d.) Babinski sign. RATIONALE: A positive Babinski sign involves dorsiflexion of the big toe and fanning of the other toes. Although normal in infants, this response is abnormal after about age 1 year or when walking begins. The stepping reflex occurs when an infant is held as though weight bearing with feet on a surface and the infant steps along raising one foot at a time. A plantar grasp reflex is characterized by flexion of the toes when a finger is placed against the base of the toes. A normal Galant reflex is initiated by stroking an infant's back alongside the spine. The hips should move toward the stimulated side. NURSING PROCESS STEP: Analysis CLIENT NEEDS CATEGORY: Health promotion and maintenance CLIENT NEEDS SUBCATEGORY: None ****************************** --> QUESTION NUMBER _ 367 _ about (MC) QUESTION: "Before surgery, a neonate is to receive an intramuscular injection of an antibiotic. Which of the following gauges and sizes of needle would the nurse select?" CHOICES ( X ) a.) 19-gauge, 1.5-inch needle. ( X ) b.) 20-gauge, 1-inch needle. ( X ) c.) 23-gauge, 2-inch needle. ( O ) d.) 25-gauge, 5/8-inch needle. RATIONALE: When administering an intramuscular injection to an infant, a 25- to 27-gauge, 0.5- to 1-inch long needle is appropriate. When administering an intramuscular injection to an infant, a 25- to 27-gauge, 0.5- to 1-inch long needle is appropriate. A 19-gauge, 1.5-inch needle is too large for an infant. When administering an intramuscular injection to an infant, a 25- to 27-gauge, 0.5- to 1-inch long needle is appropriate. A 20-gauge needle is too large for an infant. When administering an intramuscular injection to an infant, a 25- to 27-gauge, 0.5- to 1-inch long needle is appropriate. A 23-gauge, 2-inch needle is too large for an infant. NURSING PROCESS STEP: Implementation CLIENT

31

NEEDS CATEGORY: Physiological integrity CLIENT NEEDS SUBCATEGORY: Pharmacological and parenteral therapies ****************************** --> QUESTION NUMBER _ 368 _ about (MC) NB QUESTION: "When administering an intramuscular injection to a neonate, which of the following muscles would the nurse consider as the best injection site?" CHOICES ( X ) a.) Deltoid. ( X ) b.) Dorsogluteal. ( X ) c.) Ventrogluteal. ( O ) d.) Vastus lateralis. RATIONALE: The vastus lateralis muscle of the thigh is preferred for administering intramuscular injections to infants because there is less danger of injuring nerves, blood vessels, or bony structures at this site. The deltoid muscle is used for intramuscular injections only when other areas are unavailable. The dorsogluteal site is contraindicated for use in children who have not been walking for at least 1 year. The ventrogluteal site is relatively free of major nerves and blood vessels, but the vastus lateralis remains the preferred intramuscular injection site in infants. NURSING PROCESS STEP: Implementation CLIENT NEEDS CATEGORY: Physiological integrity CLIENT NEEDS SUBCATEGORY: Pharmacological and parenteral therapies ****************************** --> QUESTION NUMBER _ 369 _ about (MC) QUESTION: "Which of the following serum electrolyte levels would the nurse expect to find in an infant with persistent vomiting?" CHOICES ( O ) a.) K +, 3.2 mEq/L; Cl -, 92 mEq/L; Na +, 120 mEq/L. ( X ) b.) K +, 3.4 mEq/L; Cl -, 120 mEq/L; Na +, 140 mEq/L. ( X ) c.) K +, 3.5 mEq/L; Cl -, 90 mEq/L; Na +, 145 mEq/L. ( X ) d.) K +, 5.5 mEq/L; Cl -, 110 mEq/L; Na +, 130 mEq/L. RATIONALE: The serum electrolyte values in an infant with persistent vomiting reflect hypokalemia (K level of 3.2), hypochloremia (Cl level of 92), and hyponatremia (Na level of 120). Chloride and sodium function together to maintain fluid and electrolyte balance. With vomiting, sodium chloride and water are lost in gastric fluid. As dehydration occurs, potassium moves into the extracellular fluid. With persistent vomiting, hypokalemia (from movement of potassium into the extracellular fluid), hypochloremia (due to increased losses in gastric fluid), and hyponatremia (due to increased losses in gastric fluid ) would result. Here, the potassium level is almost normal (normal is 3.5 to 5.5 mEq/L), chloride is elevated (normal is 98 to 106 mEq/L), and sodium is normal (normal is 135 to 145 mEq/L). With persistent vomiting, hypokalemia (from movement of potassium into the extracellular fluid), hypochloremia (due to increased losses in gastric fluid), and hyponatremia (due to increased losses in gastric fluid ) would result. Here, the potassium level is normal (normal is 3.5 to 5.5 mEq/L), chloride is decreased (normal is 98 to 106 mEq/L), and sodium is normal (normal is 135 to 145 mEq/L). With persistent vomiting, hypokalemia (from movement of potassium into the extracellular fluid), hypochloremia (due to increased losses in gastric fluid), and hyponatremia (due to increased losses in gastric fluid ) would result. Here, the potassium level is normal (normal is

32

3.5 to 5.5 mE/L), chloride is slightly elevated (normal is 98 to 106 mEq/L), and sodium is slightly decreased (normal is 135 to 145 mEq/L). NURSING PROCESS STEP: Assessment CLIENT NEEDS CATEGORY: Physiological integrity CLIENT NEEDS SUBCATEGORY: Reduction of risk potential ****************************** --> QUESTION NUMBER _ 370 _ about (MC) QUESTION: "For the child experiencing excessive vomiting secondary to pyloric stenosis, the nurse should assess the child for which of the following acid:base imbalances?" CHOICES ( X ) a.) Respiratory alkalosis. ( X ) b.) Respiratory acidosis. ( O ) c.) Metabolic alkalosis. ( X ) d.) Metabolic acidosis. RATIONALE: Metabolic alkalosis occurs because of the excessive loss of potassium, hydrogen, and chloride in the vomitus. Chloride loss leads to a compensatory increase in the number of bicarbonate ions. The bicarbonate side of the carbonic acid:base bicarbonate is increased, and the pH becomes more alkaline. Respiratory alkalosis is caused by conditions such as hyperventilation that result in loss of PaCO 2. Respiratory acidosis is caused by conditions such as inadequate ventilation that result in excessive retention of PaCO 2. Metabolic acidosis results from the loss of large amounts of bicarbonate such as with severe diarrhea. NURSING PROCESS STEP: Assessment CLIENT NEEDS CATEGORY: Physiological adaptation CLIENT NEEDS SUBCATEGORY: Reduction of risk potential ****************************** --> QUESTION NUMBER _ 371 _ about (MC) QUESTION: "The infant's skin is inelastic and the upper abdomen is distended. To palpate the olive-like mass most easily, the nurse palpates the epigastrium just to the right of the umbilicus at which of the following times?" CHOICES ( X ) a.) Just before the infant vomits. ( O ) b.) While the infant is eating. ( X ) c.) When infant is lying on the left side. ( X ) d.) When the stomach is empty. RATIONALE: The pyloric olive-like mass is most easily palpated when the abdominal muscles are relaxed, the stomach is empty, and the infant is quiet. During eating, the stomach still is empty and the infant is relaxed and comfortable. The pyloric olive-like mass is most easily palpated when the abdominal muscles are relaxed, the stomach is empty, and the infant is quiet. Just before vomiting, the infant's stomach would be distended. The pyloric olive-like mass is most easily palpated when the abdominal muscles are relaxed, the stomach is empty, and the infant is quiet. The side-lying position may interfere with accurate palpation. Typically the child should be supine. The pyloric olive-like mass is most easily palpated when the abdominal muscles are relaxed, the stomach is empty, and the infant is quiet. Although the infant's stomach is empty, the infant may be crying in hunger, rather than relaxed. NURSING PROCESS STEP: Assessment CLIENT NEEDS CATEGORY: Physiological integrity CLIENT NEEDS SUBCATEGORY: Physiological adaptation ******************************

33

--> QUESTION NUMBER _ 372 _ about (MC) QUESTION: "After teaching the parents of an infant who has had a pyloromyotomy about proper postoperative feeding techniques, the nurse determines that they have understood the teaching when they position the infant in the crib after feeding with head elevated and lying on which of the following?" CHOICES ( X ) a.) Left side. ( X ) b.) Abdomen. ( O ) c.) Right side. ( X ) d.) Back. RATIONALE: Positioning the infant on the right side with the head elevated facilitates passage of food through the pyloric sphincter into the intestine. Lying on the left side would inhibit the passage of food through the pyloric sphincter into the intestine. Positioning the infant on the abdomen is associated with an increased risk for sudden infant death syndrome (SIDS). Positioning the infant on the back is associated with an increased risk for sudden infant death syndrome (SIDS). NURSING PROCESS STEP: Evaluation CLIENT NEEDS CATEGORY: Physiological integrity CLIENT NEEDS SUBCATEGORY: Reduction of risk potential ****************************** --> QUESTION NUMBER _ 373 _ about (MC) QUESTION: "After having surgery to reduce the invagination of intussusception, the infant has a nasogastric tube in place, is receiving intravenous fluids, and is allowed nothing by mouth. In addition to body weight, which of the following parameters would the nurse use to calculate the amount of intravenous fluid and electrolyte solution to be infused over the next 24 hours?" CHOICES ( X ) a.) Stool output. ( X ) b.) Urine output. ( O ) c.) Gastric output. ( X ) d.) Degree of temperature elevation. RATIONALE: The volume of parenteral fluids needed is based on fluid requirements determined according to body weight and, in this situation, gastric output. If these fluids are not replaced with an appropriate intravenous solution, serious fluid and electrolyte imbalances could develop. Stool output is monitored but it is not used to calculate maintenance and replacement needs. Although urine output is monitored, it is not used to calculate maintenance and replacement needs. Temperature is monitored but it is not used to calculate maintenance and replacement needs. NURSING PROCESS STEP: Assessment CLIENT NEEDS CATEGORY: Physiological integrity CLIENT NEEDS SUBCATEGORY: Pharmacological and parenteral therapies ****************************** --> QUESTION NUMBER _ 374 _ about (MC) QUESTION: "The mother, concerned about her infant's surgery for inguinal hernia repair, asks the nurse if her infant

34

would have been scheduled for surgery even if the hernia had been asymptomatic. Which of the following statements offers the best explanation why the surgical repair would be done at this time?" CHOICES ( X ) a.) An infant is better able to tolerate the physical stress of surgery than an older child is. ( X ) b.) The experience of surgery is less frightening for the younger child. ( O ) c.) Less danger and fewer complications result when surgery is an elective procedure. ( X ) d.) Doing surgery near the genital organs is preferred before a child becomes conscious of sexual identity. RATIONALE: Inguinal hernia repair is ordinarily done promptly after diagnosis in healthy infants and children. Delaying surgery may result in a possible partial obstruction due to a loop of bowel protruding into the inguinal canal. Serious progression with complete obstruction and perhaps strangulation of the bowel requires emergency surgery to prevent gangrene, which could be fatal. The infant does not have a physiologic or psychological advantage related to surgery when compared with older children. Infants, like any other child or adult, experience stress and fear when having surgery. Although performing surgery around the genitals before the preschool years is recommended, the best reason for performing this surgery now would be to avoid having to perform emergency surgery later. NURSING PROCESS STEP: Implementation CLIENT NEEDS CATEGORY: Physiological integrity CLIENT NEEDS SUBCATEGORY: Physiological adaptation ****************************** --> QUESTION NUMBER _ 375 _ about (MC) QUESTION: "The mother of a 7-month-old infant who has had an inguinal herniorrhaphy and is fully recovered from anesthesia asks the nurse when they will be able to leave the day surgery unit. Which of the following will the nurse use as a criterion for discharge?" CHOICES ( X ) a.) Cough reflex. ( X ) b.) Bowel movement. ( X ) c.) Systolic blood pressure of 90 mm Hg. ( O ) d.) Ability to retain an oral feeding. RATIONALE: Before discharge, the infant must be completely recovered from the anesthesia and be able to take and retain an oral feeding. Before discharge, the infant must be completely recovered from the anesthesia and be able to take and retain an oral feeding. Evidence of a cough reflex does not ensure that an infant can retain an oral feeding. Before discharge, the infant must be completely recovered from the anesthesia and be able to take and retain an oral feeding. A bowel movement may not occur for 24 hours. Thus, it is not a criteria for discharge. A normal systolic blood pressure reading for a 7-month-old infant is about 116 mm Hg. A reading of 90 mm Hg would be low, suggesting a potential problem. NURSING PROCESS STEP: Assessment CLIENT NEEDS CATEGORY: Physiological integrity CLIENT NEEDS SUBCATEGORY: Physiological adaptation ****************************** --> QUESTION NUMBER _ 376 _ about (MC) QUESTION: "A 7-month-old female infant is admitted to the hospital with a tentative diagnosis of Hirschsprung's disease. When obtaining the infant's initial health history from the parents, which of the following statements made by the mother would be most important?"

35

CHOICES ( O ) a.) "She gets constipated often." ( X ) b.) "Sometimes she gets colds." ( X ) c.) "She spits up occasionally." ( X ) d.) "Her rectal temperature is 99.4%F (37.4%C)." RATIONALE: Infants with Hirschsprung's disease typically have a history of abdominal distention, constipation, periodic diarrhea (when liquid stool leaks around the semiobstructed colon), and failure to thrive. Having an occasional cold is not unusual for an infant and is not related to Hirschsprung's disease. Spitting up once in a while is not unusual for an infant. A rectal temperature of 99.4%F would be considered normal. NURSING PROCESS STEP: Assessment CLIENT NEEDS CATEGORY: Physiological integrity CLIENT NEEDS SUBCATEGORY: Physiological adaptation ****************************** --> QUESTION NUMBER _ 377 _ about (MC) QUESTION: "After undergoing a barium enema, which of the following indicates that the infant has adequately evacuated the barium?" CHOICES ( X ) a.) Absence of fecal mass in the lower abdomen. ( O ) b.) Stools that progress from clay-colored to brown. ( X ) c.) Bowel sounds of 30 per minute. ( X ) d.) Stool guaiac that is negative. RATIONALE: The presence of barium produces white or clay-colored stools. A change in stool color from clay to normal brown is an indication that the barium has been evacuated. Presence or absence of a fecal mass does not give definitive information about the passage or retention of barium. Bowel sounds of 30 per minute suggest normal functioning but do not necessarily indicate passage of barium. A stool guaiac test is done to determine the presence of occult blood, not barium. NURSING PROCESS STEP: Evaluation CLIENT NEEDS CATEGORY: Physiological integrity CLIENT NEEDS SUBCATEGORY: Reduction of risk potential ****************************** --> QUESTION NUMBER _ 378 _ about (MC) Dosage QUESTION: "Before bowel surgery, the infant is to receive oral neomycin for 3 days. The appropriate pediatric dosage of neomycin sulfate is 10.3 mg/kg q 4 hours. The infant weighs 15 pounds, 6.4 ounces. Which of the following dosages most closely approximates a safe daily dose?" CHOICES ( X ) a.) 50 mg/day. ( X ) b.) 150 mg/day. ( X ) c.) 280 mg/day.

36

( O ) d.) 430 mg/day. RATIONALE: The dose is calculated by first determining the weight in kilograms: 15 pounds, 6.4 ounces is equal to 15.4 pounds; 15.4 pounds divided by 2.2 is equal to 7 kg. Multiplying 10.3 mg by 7 kg equals 72.1 mg. In computing a daily dosage, the dose is multiplied by the number of times it will be given per day (72.1 mg x 6 = 432.6 mg/day). The dose is calculated by first determining the weight in kilograms: 15 pounds, 6.4 ounces is equal to 15.4 pounds; 15.4 pounds divided by 2.2 is equal to 7 kg. Multiplying 10.3 mg by 7 kg equals 72.1 mg. In computing a daily dosage, the dose is multiplied by the number of times it will be given per day (72.1 mg x 6 = 432.6 mg/day). The dose is calculated by first determining the weight in kilograms: 15 pounds, 6.4 ounces is equal to 15.4 pounds; 15.4 pounds divided by 2.2 is equal to 7 kg. Multiplying 10.3 mg by 7 kg equals 72.1 mg. In computing a daily dosage, the dose is multiplied by the number of times it will be given per day (72.1 mg x 6 = 432.6 mg/day). The dose is calculated by first determining the weight in kilograms: 15 pounds, 6.4 ounces is equal to 15.4 pounds; 15.4 pounds divided by 2.2 is equal to 7 kg. Multiplying 10.3 mg by 7 kg equals 72.1 mg. In computing a daily dosage, the dose is multiplied by the number of times it will be given per day (72.1 mg x 6 = 432.6 mg/day). NURSING PROCESS STEP: Implementation CLIENT NEEDS CATEGORY: Physiological integrity CLIENT NEEDS SUBCATEGORY: Pharmacological and parenteral therapies ****************************** --> QUESTION NUMBER _ 379 _ about (MC) QUESTION: "Which of the following discharge instructions should the nurse give the parents of an infant with a temporary colostomy?" CHOICES ( X ) a.) Flush the stoma with tap water at least once a day. ( X ) b.) Allow the diaper to absorb the colostomy drainage. ( O ) c.) Give the infant plenty of liquids to drink. ( X ) d.) Expect the stoma to become dusky red within 2 weeks. RATIONALE: Because of decreased fluid reabsorption from the colon, the child with a colostomy benefits from a liberal fluid intake. Infants also dehydrate more quickly than adults do because of immature kidneys, larger body surface area, and more fluid in the extracellular spaces. Therefore, the parents need instructions about giving the infant plenty of liquids to drink. Tap water flushes of the stoma are contraindicated in infants because of the risk for absorption of free water and potential for fluid overload. An appliance should be fitted over the stoma for stool collection to help prevent skin breakdown. The stoma should always be reddish-pink and moist. A dusky-red stoma may indicate impaired circulation to the area. NURSING PROCESS STEP: Implementation CLIENT NEEDS CATEGORY: Physiological integrity CLIENT NEEDS SUBCATEGORY: Physiological adaptation ****************************** --> QUESTION NUMBER _ 380 _ about (MC) Dosage QUESTION: "A 15-month-old child with an intravenous line in place is ordered to receive a total of 200 mL over the next 3 hours. The infusion set delivers 1 mL per 60 drops. At which of the following rates would the nurse run the infusion?" CHOICES ( X ) a.) 14 drops/minute. ( X ) b.) 21 drops/minute. ( O ) c.) 67 drops/minute.

37

( X ) d.) 83 drops/minute. RATIONALE: The drop rate is determined as follows: 200 mL fluid/3 hours = 67 mL (rounded from 66.67) fluid/1 hour 67 mL x 60 drops/mL = 4020 drops 4020 drops divided by 60 minutes = 67 drops/minute. The drop rate is determined as follows: 200 mL fluid/3 hours = 67 mL (rounded from 66.67) fluid/1 hour 67 mL x 60 drops/mL = 4020 drops 4020 drops divided by 60 minutes = 67 drops/minute. The drop rate is determined as follows: 200 mL fluid/3 hours = 67 mL (rounded from 66.67) fluid/1 hour 67 mL x 60 drops/mL = 4020 drops 4020 drops divided by 60 minutes = 67 drops/minute The drop rate is determined as follows: 200 mL fluid/3 hours = 67 mL (rounded from 66.67) fluid/1 hour 67 mL x 60 drops/mL = 4020 drops 4020 drops divided by 60 minutes = 67 drops/minute NURSING PROCESS STEP: Implementation CLIENT NEEDS CATEGORY: Physiological integrity CLIENT NEEDS SUBCATEGORY: Pharmacological and parenteral therapies ****************************** --> QUESTION NUMBER _ 381 _ about (MC) QUESTION: "An 11-month-old infant is admitted to the hospital with severe diarrhea. The nurse explains to the infant's mother that diarrhea is best defined on the basis of which of the following stool characteristics?" CHOICES ( X ) a.) Odor. ( X ) b.) Amount. ( X ) c.) Frequency. ( O ) d.) Consistency. RATIONALE: Diarrhea is best defined on the basis of stool consistency that is commonly liquid in nature. The color of diarrheal stools is usually greenish, but stool color is also affected by food and fluid intake. Stool odor is not directly related to diarrhea. It can vary widely and is not an accurate criterion to define diarrhea. Estimates of the amount of stool can vary widely. Therefore, this is not an accurate criterion by which to define diarrhea. The frequency of stools varies, although stools occur more frequently than normal when diarrhea is present. NURSING PROCESS STEP: Implementation CLIENT NEEDS CATEGORY: Physiological integrity CLIENT NEEDS SUBCATEGORY: Physiological adaptation ****************************** --> QUESTION NUMBER _ 382 _ about (MC) QUESTION: "For the child who exhibits signs of discomfort and swelling in the region where an intravenous needle is inserted, the nurse would suspect which of the following?" CHOICES ( O ) a.) Needle has come out of the vein. ( X ) b.) Intravenous site has been used too long. ( X ) c.) Child is allergic to the plastic in the needle. ( X ) d.) Rate of fluid administration is too rapid for the vein size. RATIONALE: Pain and swelling in the area of needle insertion most likely indicate that the needle has come out of the

38

vein. Swelling occurs as the fluid infuses into subcutaneous tissues. Other typical signs of infiltration include skin pallor and coldness around the insertion site. Signs of inflammation, such as redness and warmth, are likely if the intravenous site is used too long. Because inert plastic is used for manufacturing intravenous needles, the risk of an allergic reaction is remote. If fluid is administered too rapidly for the vein size, the fluid would most probably leak around the needle at the area of connection with the tubing. NURSING PROCESS STEP: Analysis CLIENT NEEDS CATEGORY: Physiological integrity CLIENT NEEDS SUBCATEGORY: Pharmacological and parenteral therapies ****************************** --> QUESTION NUMBER _ 383 _ about (MC) Appendicitis QUESTION: "A 14-year-old boy brought to the emergency department complaining of right lower quadrant pain is tentatively diagnosed with acute appendicitis. When assessing the client, which of the following would the nurse expect to find?" CHOICES ( X ) a.) Costovertebral angle tenderness. ( X ) b.) Widening pulse pressure. ( O ) c.) Oral temperature of 100%F (37.8%C). ( X ) d.) Gross hematuria. RATIONALE: The most common manifestations of appendicitis include right lower quadrant pain, localized tenderness, and fever of 99%F (37.2%C) to 102%F (38.9%C). Other signs of inflammation including increased pulse and respiratory rates may be present. Costovertebral angle tenderness and hematuria are associated with urologic problems. Widening pulse pressure is seen in increased intracranial pressure. Gross hematuria and costovertebral angle tenderness are associated with urologic problems. NURSING PROCESS STEP: Assessment CLIENT NEEDS CATEGORY: Physiological integrity CLIENT NEEDS SUBCATEGORY: Physiological adaptation ****************************** --> QUESTION NUMBER _ 384 _ about (MC) Appendicitis QUESTION: "After an appendectomy, an adolescent client is alert and oriented. Parenteral fluids are infusing and a nasogastric tube is attached to low intermittent suction. Which of the following nursing measures would be most appropriate for the client during this early postoperative period?" CHOICES ( X ) a.) Irrigating the nasogastric tube every hour. ( X ) b.) Testing the urine for protein. ( X ) c.) Removing the nasogastric tube when fully alert. ( O ) d.) Encouraging the client to urinate frequently. RATIONALE: After an appendectomy, the client should be encouraged to void frequently to prevent bladder distention which could cause strain on the incision. There is no reason to irrigate the nasogastric tube unless it ceases to function or the drainage is thick and tenacious. There is no indication that the client has a renal problem. Therefore, testing urine for protein is unnecessary. The nasogastric tube remains in place until peristalsis returns, evidenced by active bowel sounds and decreasing amounts of drainage. NURSING PROCESS STEP: Implementation CLIENT NEEDS CATEGORY: Physiological integrity CLIENT NEEDS SUBCATEGORY: Physiological adaptation ******************************

39

--> QUESTION NUMBER _ 385 _ about (MC) Drug poisoning QUESTION: "A mother brings her child to the emergency room after the child has taken "some white pills just a short while ago." Which of the following would lead the nurse to determine that the pills taken were most probably acetaminophen?" CHOICES ( X ) a.) Nosebleed. ( X ) b.) Seizure activity. ( O ) c.) Nausea and vomiting. ( X ) d.) Deep rapid respirations. RATIONALE: Acetaminophen is a common drug poisoning agent in children. Symptoms seen in the first 24 hours include nausea and vomiting, anorexia, malaise, and pallor. Nosebleed and deep, rapid respirations are seen in salicylate poisoning. Seizure activity is not commonly seen with acetaminophen poisoning. Deep rapid respirations and nosebleed are seen in salicylate poisoning. NURSING PROCESS STEP: Assessment CLIENT NEEDS CATEGORY: Physiological integrity CLIENT NEEDS SUBCATEGORY: Physiological adaptation ****************************** --> QUESTION NUMBER _ 386 _ about (MC) QUESTION: "Which of the following assessment findings would the nurse expect in an infant with colic?" CHOICES ( X ) a.) Failure to gain weight. ( O ) b.) Expulsion of flatus. ( X ) c.) Soft abdomen. ( X ) d.) Difficulty with burping. RATIONALE: Infants with colic have paroxysmal abdominal pain or cramping caused by the production and accumulation of gas. This causes pain and abdominal distention. They may expel flatus or eructate but they do not vomit. Despite their pain, infants with colic typically tolerate formula well, gain weight, and thrive. Infants with colic typically exhibit a distended abdomen from the accumulation of gas. Typically infants with colic do not experience difficulty with burping. NURSING PROCESS STEP: Assessment CLIENT NEEDS CATEGORY: Physiological integrity CLIENT NEEDS SUBCATEGORY: Physiological adaptation ****************************** --> QUESTION NUMBER _ 387 _ about (MC) QUESTION: "The adolescent client tells the nurse that he would like to lose weight and asks the nurse's opinion on how to accomplish his goal. Which of the following suggestions would be most appropriate? " CHOICES ( X ) a.) Exercising more often.

40

( X ) b.) Severely limiting calorie intake. ( O ) c.) Participating in an adolescent weight-reduction program. ( X ) d.) Cutting down on sweets and other snacks. RATIONALE: Weight loss treatment modalities that include peer involvement have been proven to be the most successful approach with obese adolescents. This is because peer support is critical to adolescents, especially with an all-encompassing problem such as obesity. Increasing the amount of exercise is helpful, but this is just one aspect of a weight reduction program. Severe calorie restriction is not recommended because it can result in use of muscle protein as well as fat for energy. Although decreased ingestion of nonnutritive snacks is helpful in dietary control, there is no evidence that this is a problem for this adolescent. NURSING PROCESS STEP: Implementation CLIENT NEEDS CATEGORY: Health promotion and maintenance CLIENT NEEDS SUBCATEGORY: None ****************************** --> QUESTION NUMBER _ 388 _ about (MC) Postop QUESTION: "A 14-month-old child returns from surgery for undescended testicle, and his postanesthesia recovery period is uneventful. When planning for the child's discharge, which of the following goals would the nurse expect to emphasize to the parents?" CHOICES ( O ) a.) Absence of redness or swelling at the incision site. ( X ) b.) Intake clear liquids well within 24 hours. ( X ) c.) Passage of normal bowel movement within 24 hours. ( X ) d.) Ability to ambulate after 48 hours. RATIONALE: As with any surgery or invasive procedure, a priority goal at this time would be to prevent infection at the operative site. The child can usually begin to take fluids and solids shortly after surgery and should be able to tolerate them prior to discharge. Defecation is not a usual problem after this type of surgery because the bowel is not involved. Usually the child can get up as soon as comfort allows. NURSING PROCESS STEP: Planning CLIENT NEEDS CATEGORY: Physiological integrity CLIENT NEEDS SUBCATEGORY: Physiological adaptation ****************************** --> QUESTION NUMBER _ 389 _ about (MC) QUESTION: "After teaching the mother of a 3-year-old girl about measures to help prevent urinary tract infections, which of the following statements by the mother indicates successful teaching?" CHOICES ( X ) a.) "She'll love the idea of taking more bubble baths." ( O ) b.) "We'll buy some of her favorite juice so she'll drink more." ( X ) c.) "We'll try to get her not to go to the bathroom too frequently." ( X ) d.) "We'll let her soak in the bathtub for 30 minutes every day."

41

RATIONALE: Increased fluid intake promotes frequent urination, which flushes bacteria out of the urinary tract. Having favorite juices available for the child increases the chances for success by encouraging the child to drink. Bubble baths and long soaks in the tub may result in irritation, possibly resulting in painful urination and subsequent inadequate bladder emptying. Emptying the bladder frequently and at the first urge to void prevents urinary stasis and decreases the risk of infection ascending to the kidneys. Long soaks in the tub may result in irritation, possibly resulting in painful urination and subsequent inadequate bladder emptying. NURSING PROCESS STEP: Evaluation CLIENT NEEDS CATEGORY: Health promotion and maintenance CLIENT NEEDS SUBCATEGORY: None ****************************** --> QUESTION NUMBER _ 390 _ about (MC) QUESTION: "Which of the following would lead the nurse to suspect that a school-aged child is experiencing a severe complication of acute glomerulonephritis?" CHOICES ( X ) a.) Temperature of 100.2%F (38.8%C). ( X ) b.) Serum sodium level of 135 mEq/L. ( O ) c.) Blood pressure of 140/92 mm Hg. ( X ) d.) Weight loss of 2 pounds. RATIONALE: The elevated blood pressure may indicate hypertension, which is a serious complication of acute glomerulonephritis. The temperature is only slightly elevated and may be related to the glomerulonephritis infection. The serum sodium level is in the normal range. The weight loss is probably due to a loss of fluid that the client had been retaining. NURSING PROCESS STEP: Analysis CLIENT NEEDS CATEGORY: Physiological integrity CLIENT NEEDS SUBCATEGORY: Physiological adaptation ****************************** --> QUESTION NUMBER _ 391 _ about (MC) QUESTION: "A 3-year-old child hospitalized for observation has marked dependent edema and hypoalbuminemia. His urine is frothy but he is free from infection. When assessing the child's vital signs, the nurse would expect to find which of the following?" CHOICES ( O ) a.) Blood pressure of 100/60 mm Hg. ( X ) b.) Body temperature of 100.8%F (38.2%C). ( X ) c.) Pulse rate of 72 bpm. ( X ) d.) Respiratory rate of 18 breaths/minute. RATIONALE: The child is displaying signs and symptoms of nephrotic syndrome. With this disorder, blood pressure is characteristically normal or slightly low. The other vital signs are likely to be normal unless edema causes respiratory distress and the respirations increase and become labored. The blood pressure reading here is within the normal range for a 3-year-old child. Temperature of 100.8%F (38.2%C) is elevated, suggesting an infection. However, the scenario states that the child is free from infection. A pulse rate of 72 bpm is low for a 3-year-old child. In nephrotic syndrome, the pulse rate would be normal unless other problems arise. A respiratory rate of 18 is low for a 3-year-old child. In nephrotic syndrome, the respiratory rate would be normal unless edema causes respiratory distress and the respirations increase and become labored. NURSING PROCESS STEP: Analysis CLIENT NEEDS CATEGORY: Physiological

42

integrity CLIENT NEEDS SUBCATEGORY: Physiological adaptation ****************************** --> QUESTION NUMBER _ 392 _ about (MC) QUESTION: "Which of the following nursing measures would the nurse institute to help reduce eyelid edema in a child with nephrotic syndrome?" CHOICES ( X ) a.) Apply cool compresses to the child's eyes. ( O ) b.) Elevate the head of the child's bed. ( X ) c.) Instill eye drops every 8 hours. ( X ) d.) Limit the child's television watching. RATIONALE: Elevating the head of the bed allows gravity to increase the downward flow of fluids in the body and away from the face. Applying cool compresses may be comforting but will not help to decrease edema. Instilling eye drops may be comforting but will not help to decrease edema. Limiting television may be comforting but will not decrease edema. NURSING PROCESS STEP: Implementation CLIENT NEEDS CATEGORY: Physiological integrity CLIENT NEEDS SUBCATEGORY: Physiological adaptation ****************************** --> QUESTION NUMBER _ 393 _ about (MC) QUESTION: "A male neonate with a 3 cm-by-5 cm sac in the lumbar region of his back is diagnosed with myelomeningocele. Which of the following would the nurse expect to find when inspecting this sac?" CHOICES ( X ) a.) Serosanguineous fluid and fatty tissue. ( X ) b.) Bits of hair covered by skin. ( X ) c.) Spinal fluid and meninges. ( O ) d.) Spinal fluid, nerve tissue, and spinal bony defect. RATIONALE: A myelomeningocele has three components (bony defect, spinal fluid, and nerve tissue such as meninges, spinal cord, or nerve roots) and protrudes over the vertebrae, usually in the lower back. A simple cyst contains serosanguineous fluid and fatty tissue located on any area of the spinal column. A pilonidal cyst is a skin-covered sac containing bits of hair located on the low lumbar or sacral area of the spine. A meningocele is a soft sac containing only spinal fluid and meninges located anywhere on the spine. NURSING PROCESS STEP: Assessment CLIENT NEEDS CATEGORY: Physiological integrity CLIENT NEEDS SUBCATEGORY: Physiological adaptation ****************************** --> QUESTION NUMBER _ 394 _ about (MC) QUESTION: "Which of the following should the nurse do first when a neonate with myelomeningocele experiences urine retention with overflow incontinence?" CHOICES

43

( X ) a.) Apply pressure to the suprapubic area. ( O ) b.) Initiate an intermittent clean catheterization program. ( X ) c.) Insert an indwelling urinary catheter. ( X ) d.) Collect a urine specimen. RATIONALE: Overflow incontinence with constant dribbling is common in neonates with myelomeningocele. Intermittent clean catheterization is an appropriate technique for management of urinary retention or when a urine specimen is urgently needed. Applying pressure to the suprapubic area (location of the bladder) is not done because it can create high pressure in the bladder and can possibly force urine back up the ureters, increasing the risk for infection. Inserting an indwelling urinary catheter is done most frequently when the neonate is unable to void. Collecting a urine specimen provides urine for analysis but will do nothing to aid in relieving overflow incontinence. NURSING PROCESS STEP: Implementation CLIENT NEEDS CATEGORY: Physiological integrity CLIENT NEEDS SUBCATEGORY: Physiological adaptation ****************************** --> QUESTION NUMBER _ 395 _ about (MC) QUESTION: "Which of the following would the nurse include when developing the plan of care for a neonate prior to surgical repair of a myelomeningocele?" CHOICES ( X ) a.) Applying thin layers of tincture of benzoin around the defect. ( X ) b.) Positioning the neonate on the side. ( O ) c.) Covering the defect with moist, sterile saline dressings. ( X ) d.) Leaving the defect exposed to air. RATIONALE: The sac is kept moist by covering it with nonadherent, sterile saline dressings. The dressings will need to be moistened often to prevent them from drying out. The sac also is inspected carefully for leaks, abrasions, and signs of infection. Tincture of benzoin is an adherent and would not be used because it could potentially cause disruption of the neonate's skin integrity. The neonate should be positioned on the abdomen to avoid tearing the sac. The sac must be kept moist. If left open to the air, it would dry out, possibly causing the sac to tear which would allow cerebrospinal fluid to leak. NURSING PROCESS STEP: Planning CLIENT NEEDS CATEGORY: Physiological integrity CLIENT NEEDS SUBCATEGORY: Physiological adaptation ****************************** --> QUESTION NUMBER _ 396 _ about (MC) Hydrocephalus QUESTION: "Assessment of a 6-week-old infant reveals weight and length in the 50th percentile for his age and a head circumference at the 95th percentile. Which of the following would the nurse do first?" CHOICES ( X ) a.) Assess motor and sensory function of the legs. ( O ) b.) Examine the fontanels and sutures. ( X ) c.) Advise the mother of the need for follow-up in 1 month.

44

( X ) d.) Obtain a written consent for transillumination. RATIONALE: Head circumference usually parallels the percentile for length. The discrepancy found requires close and immediate attention because it could indicate hydrocephalus with its potential for brain damage. Therefore, the nurse should examine the fontanels and sutures. In an infant, bulging fontanels and widening cranial sutures are signs of increasing intracranial pressure related to increased cerebrospinal fluid in the cranial space. Assessing motor and sensory function of the legs would be done if the fontanel or sutures were abnormal. Head circumference usually parallels the percentile for length. The discrepancy found requires close and immediate attention, not follow-up in 1 month, because it could indicate hydrocephalus with its potential for brain damage. Transillumination is a noninvasive procedure used to assess hydrocephalus. It does not require a written consent and would be performed after examining the fontanels and sutures. NURSING PROCESS STEP: Implementation CLIENT NEEDS CATEGORY: Health promotion and maintenance CLIENT NEEDS SUBCATEGORY: None ****************************** --> QUESTION NUMBER _ 397 _ about (MC) QUESTION: "Which of the following nursing interventions is most important postoperatively for an infant who has received a ventriculoperitoneal shunt?" CHOICES ( O ) a.) Monitoring intake and output. ( X ) b.) Initiating oral feedings. ( X ) c.) Allowing the infant to rest undisturbed. ( X ) d.) Providing age-appropriate diversionary activities. RATIONALE: In the postoperative period, intake and output are carefully monitored to prevent fluid overload that could lead to increased intracranial pressure. Feedings will be started when the infant is fully awake. The infant will need to be disturbed to check vital signs and be repositioned. Age-appropriate activities are important but not until the infant is awake and less fussy. NURSING PROCESS STEP: Implementation CLIENT NEEDS CATEGORY: Physiological integrity CLIENT NEEDS SUBCATEGORY: Reduction of risk potential ****************************** --> QUESTION NUMBER _ 398 _ about (MC) Seizure QUESTION: "A girl in second grade with no remarkable medical history experiences a generalized tonic:clonic seizure in the classroom. Immediately after the seizure, the nurse arrives and notices that the child has been incontinent of urine and is difficult to arouse. Which of the following actions would be most appropriate at this time?" CHOICES ( X ) a.) Ask the teacher if the child has had any urinary problems. ( X ) b.) Awaken the child every 3 to 5 minutes to assess mentation. ( X ) c.) Perform a complete neurologic check every 3 to 5 minutes. ( O ) d.) Stay with the child and allow her to sleep in a side-lying position. RATIONALE: During the postictal period of a generalized tonic:clonic seizure, it is normal for a child to sleep and be

45

difficult to arouse. During this time, the nurse should stay with the child, allowing sleep until he or she awakens. The side-lying position is best to prevent possible aspiration. Sleep and drowsiness do not follow other forms of generalized seizures. During a seizure, urinary incontinence is common. Therefore, asking the teacher about any urinary problems would be inappropriate. After a tonic:clonic seizure, children are sleepy and fussy. The child should be allowed to sleep until he or she awakens. Obtaining information about neurologic status is important but awakening the child every 3 to 5 minutes would not be helpful. NURSING PROCESS STEP: Implementation CLIENT NEEDS CATEGORY: Physiological integrity CLIENT NEEDS SUBCATEGORY: Physiological adaptation ****************************** --> QUESTION NUMBER _ 399 _ about (MC) Seizure QUESTION: "When developing a teaching plan for the family of a child with seizures, which of the following would the nurse include when discussing pharmacologic treatment?" CHOICES ( X ) a.) Medication is adjusted independently when side effects occur. ( O ) b.) Abrupt cessation of the medication must be avoided. ( X ) c.) Dosages will be decreased as the child grows older. ( X ) d.) Medication therapy is necessary for the rest of the child's life. RATIONALE: Abrupt cessation of the medication must be avoided because sudden drug withdrawal most commonly leads to status epilepticus, a life-threatening emergency situation. The family should notify the physician of any troublesome side effects. The physician will determine if an adjustment in the medication is needed. Medication dosages are commonly determined by the child's weight in kilograms. Physical growth, such as during adolescence, and the accompanying increase in weight frequently necessitate a dosage increase to maintain therapeutic drug levels. Some children may be able to discontinue their medication but only under supervised conditions and after being completely seizure-free on medication for several years. NURSING PROCESS STEP: Planning CLIENT NEEDS CATEGORY: Physiological integrity CLIENT NEEDS SUBCATEGORY: Pharmacological and parenteral therapies ****************************** --> QUESTION NUMBER _ 400 _ about (MC) Seizure QUESTION: "Which of the following would the nurse do for a 4-year-old girl who has just had a lumbar puncture?" CHOICES ( X ) a.) Administer narcotic analgesic for insertion site pain. ( O ) b.) Encourage the parents to hold the child. ( X ) c.) Ensure the child lies flat for at least 8 hours. ( X ) d.) Place a sandbag over the puncture site for 3 hours. RATIONALE: After a lumbar puncture which is a traumatic procedure for a 4-year-old child, the child needs to be comforted by people she trusts. Thus, the nurse should encourage the parents to hold the child to provide the necessary support. The child should feel little pain at the insertion site after the lumbar puncture. Narcotics would not be the drugs of choice because they hinder assessment of neurologic status. A young child does not need to lay flat for any time after a lumbar puncture. Additionally, lying flat for 8 hours would be difficult, if not impossible, for a 4-year-old child. After a lumbar puncture, the application of a small bandage after applying pressure for a short time is usually sufficient to stop any leakage and prevent infection of the site. A sandbag is not needed.

46

Anda mungkin juga menyukai